0% found this document useful (0 votes)
538 views108 pages

Mastery I

This is a book to prepare junior participants in mathematics competitions.

Uploaded by

hsadec25
Copyright
© © All Rights Reserved
We take content rights seriously. If you suspect this is your content, claim it here.
Available Formats
Download as PDF, TXT or read online on Scribd
0% found this document useful (0 votes)
538 views108 pages

Mastery I

This is a book to prepare junior participants in mathematics competitions.

Uploaded by

hsadec25
Copyright
© © All Rights Reserved
We take content rights seriously. If you suspect this is your content, claim it here.
Available Formats
Download as PDF, TXT or read online on Scribd
You are on page 1/ 108

2

THE MATH OLYMPIAD MASTERY LEVEL I

1st EDITION

August 2023
2

Authors
Nkundayezu Emmanuel
Musangwa Innocent
Warakoze Bel-ami
Abayo Joseph Desire
Munezero Jean Nepo
Akimana Nadine
Korusenge Obed
Hategekimana Hirwa Arnold
Irakarama Alain
Subukino Arnaud
Kagaba Etienne

Kigali City, Rwanda

Contact: [email protected]
3

PREFACE
Welcome to ”Math Olympiad Mastery: A Comprehensive Guide to Algebra, Number Theory, Geometry,
and Combinatorics.” This book is a collaborative effort by a team of dedicated authors who are passionate
about mathematics and committed to helping you excel in math competitions and problem-solving challenges.

The authors, Nkundayezu Emmanuel, Musangwa Innocent, Warakoze Bel-ami, Abayo Joseph Desire,
Munezero Jean Nepo, Akimana Nadine, Korusenge Obed, Hategekimana Hirwa Arnold, Irakarama Alain,
Subukino Arnaud, and Kagaba Etienne, have combined their knowledge, experience, and enthusiasm to
create a comprehensive resource that will empower you to tackle the intricate and fascinating world of math-
ematical problem-solving.

In this book, you will embark on a journey through the realms of algebra, number theory, geometry, and
combinatorics - key areas of mathematical Olympiad competitions. Each chapter is meticulously crafted
to provide a deep understanding of the fundamental concepts, problem-solving techniques, and strategies
required to conquer even the most challenging problems.

Key Features of the Book:


Diverse Authorship: With a team of authors hailing from various backgrounds and perspectives, this
book offers a rich and diverse range of insights, ensuring a well-rounded approach to problem-solving.

Comprehensive Content: The book is divided into four major sections - Algebra, Number Theory, Ge-
ometry, and Combinatorics. Each section is thoughtfully structured, progressing from foundational concepts
to advanced problem-solving strategies.

Clear and Concise Explanations: Complex mathematical ideas are explained in a clear and straight-
forward manner, making even the most intricate topics accessible to readers of all levels.

Abundant Examples and Problems: Throughout the book, you will find numerous examples and care-
fully selected problems that illustrate the concepts discussed and provide ample practice opportunities.

Challenging Exercises: In each chapter, you’ll encounter a range of exercises, from standard problems
to challenging ones designed to stretch your problem-solving skills and expand your mathematical thinking.

Problem-Solving Strategies: Alongside the content, you’ll discover valuable problem-solving strategies,
tips, and techniques that will equip you to approach Olympiad-style problems with confidence and creativity.

Whether you are a novice eager to explore the world of math competitions or a seasoned problem solver
seeking to refine your skills, ”Math Olympiad Mastery” is designed to be your trusted companion. The
authors have poured their expertise into every page, aiming to empower you to overcome obstacles, unravel
complex problems, and achieve your fullest mathematical potential.

Happy problem solving!


The Authors
4
Contents

I MATH OLYMPIAD CONTENT 9


1 ALGEBRA 11
1.1 SETS AND SUBSETS . . . . . . . . . . . . . . . . . . . . . . . . . . . . . . . . . . . . . . . . 11
1.1.1 History of numbers . . . . . . . . . . . . . . . . . . . . . . . . . . . . . . . . . . . . . . 11
1.1.2 Sets and subsets . . . . . . . . . . . . . . . . . . . . . . . . . . . . . . . . . . . . . . . 12
1.2 FUNCTIONS . . . . . . . . . . . . . . . . . . . . . . . . . . . . . . . . . . . . . . . . . . . . . 16
1.2.1 Input & Output, Domain, Co-domain and Range, Injective, Surjective and Bijective
Functions . . . . . . . . . . . . . . . . . . . . . . . . . . . . . . . . . . . . . . . . . . . 16
1.2.2 Input & Output . . . . . . . . . . . . . . . . . . . . . . . . . . . . . . . . . . . . . . . 16
1.2.3 Domain, Co-domain and Range . . . . . . . . . . . . . . . . . . . . . . . . . . . . . . . 16
1.2.4 EXERCISES . . . . . . . . . . . . . . . . . . . . . . . . . . . . . . . . . . . . . . . . . 17
1.2.5 Injective, Surjective and Bijective Functions . . . . . . . . . . . . . . . . . . . . . . . . 17
1.2.6 EXERCISES . . . . . . . . . . . . . . . . . . . . . . . . . . . . . . . . . . . . . . . . . 19
1.2.7 Linear Functions . . . . . . . . . . . . . . . . . . . . . . . . . . . . . . . . . . . . . . . 20
1.2.8 Common terms associated with linear functions. . . . . . . . . . . . . . . . . . . . . . 20
1.3 INEQUALITIES . . . . . . . . . . . . . . . . . . . . . . . . . . . . . . . . . . . . . . . . . . . 24
1.3.1 Introduction . . . . . . . . . . . . . . . . . . . . . . . . . . . . . . . . . . . . . . . . . 24
1.3.2 Basic properties . . . . . . . . . . . . . . . . . . . . . . . . . . . . . . . . . . . . . . . 24
1.3.3 Examples . . . . . . . . . . . . . . . . . . . . . . . . . . . . . . . . . . . . . . . . . . . 25
1.3.4 EXERCISE A . . . . . . . . . . . . . . . . . . . . . . . . . . . . . . . . . . . . . . . . 27
1.3.5 EXERCISE B . . . . . . . . . . . . . . . . . . . . . . . . . . . . . . . . . . . . . . . . . 27
1.4 PROBLEMS . . . . . . . . . . . . . . . . . . . . . . . . . . . . . . . . . . . . . . . . . . . . . 28

2 NUMBER THEORY 33
2.1 INTEGERS: THE BASICS . . . . . . . . . . . . . . . . . . . . . . . . . . . . . . . . . . . . . 33
2.1.1 Introduction . . . . . . . . . . . . . . . . . . . . . . . . . . . . . . . . . . . . . . . . . 33
2.2 TYPES OF INTEGERS . . . . . . . . . . . . . . . . . . . . . . . . . . . . . . . . . . . . . . . 33
2.2.1 Fundamental arithmetic operations . . . . . . . . . . . . . . . . . . . . . . . . . . . . . 33
2.2.2 Problems . . . . . . . . . . . . . . . . . . . . . . . . . . . . . . . . . . . . . . . . . . . 34
2.3 EVEN AND ODD NUMBERS . . . . . . . . . . . . . . . . . . . . . . . . . . . . . . . . . . . 35
2.3.1 Definition . . . . . . . . . . . . . . . . . . . . . . . . . . . . . . . . . . . . . . . . . . . 35
2.3.2 Even and odd properties . . . . . . . . . . . . . . . . . . . . . . . . . . . . . . . . . . . 35
2.3.3 Problems . . . . . . . . . . . . . . . . . . . . . . . . . . . . . . . . . . . . . . . . . . . 36
2.4 PRIMES and COMPOSITES . . . . . . . . . . . . . . . . . . . . . . . . . . . . . . . . . . . . 36
2.4.1 Introduction . . . . . . . . . . . . . . . . . . . . . . . . . . . . . . . . . . . . . . . . . 36
2.4.2 Identifying primes . . . . . . . . . . . . . . . . . . . . . . . . . . . . . . . . . . . . . . 37
2.4.3 Prime factorization . . . . . . . . . . . . . . . . . . . . . . . . . . . . . . . . . . . . . . 38
2.4.4 Problems . . . . . . . . . . . . . . . . . . . . . . . . . . . . . . . . . . . . . . . . . . . 38
2.5 MULTIPLES and DIVISORS . . . . . . . . . . . . . . . . . . . . . . . . . . . . . . . . . . . . 39
2.5.1 Greatest common divisor . . . . . . . . . . . . . . . . . . . . . . . . . . . . . . . . . . 39

5
6 CONTENTS

2.5.2 Computing the greatest common divisor . . . . . . . . . . . . . . . . . . . . . . . . . . 39


2.5.3 Lowest common multiple (LCM) . . . . . . . . . . . . . . . . . . . . . . . . . . . . . . 42
2.5.4 Computing lowest common multiple . . . . . . . . . . . . . . . . . . . . . . . . . . . . 42
2.5.5 Properties of LCM . . . . . . . . . . . . . . . . . . . . . . . . . . . . . . . . . . . . . . 43
2.5.6 Relationship between GCD and LCM . . . . . . . . . . . . . . . . . . . . . . . . . . . 43
2.5.7 Problems . . . . . . . . . . . . . . . . . . . . . . . . . . . . . . . . . . . . . . . . . . . 43
2.5.8 Review problems . . . . . . . . . . . . . . . . . . . . . . . . . . . . . . . . . . . . . . . 43
2.5.9 Challenging problems . . . . . . . . . . . . . . . . . . . . . . . . . . . . . . . . . . . . 43
2.6 DIVISIBILITY . . . . . . . . . . . . . . . . . . . . . . . . . . . . . . . . . . . . . . . . . . . . 43
2.6.1 Divisibility rules . . . . . . . . . . . . . . . . . . . . . . . . . . . . . . . . . . . . . . . 43
2.6.2 Remainders . . . . . . . . . . . . . . . . . . . . . . . . . . . . . . . . . . . . . . . . . . 45
2.6.3 Problems . . . . . . . . . . . . . . . . . . . . . . . . . . . . . . . . . . . . . . . . . . . 45
2.7 PERFECT SQUARES and POWERS . . . . . . . . . . . . . . . . . . . . . . . . . . . . . . . 46
2.7.1 introduction . . . . . . . . . . . . . . . . . . . . . . . . . . . . . . . . . . . . . . . . . . 46
2.7.2 Perfect squares . . . . . . . . . . . . . . . . . . . . . . . . . . . . . . . . . . . . . . . . 46
2.7.3 Properties of perfect squares . . . . . . . . . . . . . . . . . . . . . . . . . . . . . . . . 46
2.7.4 Perfect powers . . . . . . . . . . . . . . . . . . . . . . . . . . . . . . . . . . . . . . . . 46
2.7.5 Problems . . . . . . . . . . . . . . . . . . . . . . . . . . . . . . . . . . . . . . . . . . . 47
2.8 PROBLEMS . . . . . . . . . . . . . . . . . . . . . . . . . . . . . . . . . . . . . . . . . . . . . 48

3 GEOMETRY 51
3.1 ANGLES AND THEIR PROPERTIES . . . . . . . . . . . . . . . . . . . . . . . . . . . . . . . 51
3.1.1 Triangle and its properties . . . . . . . . . . . . . . . . . . . . . . . . . . . . . . . . . . 52
3.2 QUADRILATERALS . . . . . . . . . . . . . . . . . . . . . . . . . . . . . . . . . . . . . . . . 57
3.2.1 Convex Quadrilaterals . . . . . . . . . . . . . . . . . . . . . . . . . . . . . . . . . . . . 57
3.2.2 Cyclic quadrilaterals . . . . . . . . . . . . . . . . . . . . . . . . . . . . . . . . . . . . . 58
3.2.3 Solved example . . . . . . . . . . . . . . . . . . . . . . . . . . . . . . . . . . . . . . . 58
3.3 CIRCLE . . . . . . . . . . . . . . . . . . . . . . . . . . . . . . . . . . . . . . . . . . . . . . . . 59
3.3.1 Tangents and its properties . . . . . . . . . . . . . . . . . . . . . . . . . . . . . . . . . 59
3.3.2 Chords and its properties . . . . . . . . . . . . . . . . . . . . . . . . . . . . . . . . . . 59
3.3.3 Circle Theorems . . . . . . . . . . . . . . . . . . . . . . . . . . . . . . . . . . . . . . . 60
3.4 INTRODUCTION TO CARTESIAN PLANE . . . . . . . . . . . . . . . . . . . . . . . . . . . 63
3.4.1 Introduction . . . . . . . . . . . . . . . . . . . . . . . . . . . . . . . . . . . . . . . . . 63
3.4.2 Key Features: . . . . . . . . . . . . . . . . . . . . . . . . . . . . . . . . . . . . . . . . . 63
3.4.3 How to find a point coordinates on cartesian plane . . . . . . . . . . . . . . . . . . . . 63
3.5 APPROACHES AND BASICS IN SOLVING OLYMPIAD PROBLEMS USING GEOMETRY 65
3.5.1 Angle chasing . . . . . . . . . . . . . . . . . . . . . . . . . . . . . . . . . . . . . . . . . 65
3.5.2 Working backwards . . . . . . . . . . . . . . . . . . . . . . . . . . . . . . . . . . . . . 69
3.6 PROBLEMS . . . . . . . . . . . . . . . . . . . . . . . . . . . . . . . . . . . . . . . . . . . . . 70

4 COMBINATORICS 73
4.1 COUNTING METHODS . . . . . . . . . . . . . . . . . . . . . . . . . . . . . . . . . . . . . . 73
4.1.1 Introduction . . . . . . . . . . . . . . . . . . . . . . . . . . . . . . . . . . . . . . . . . 73
4.1.2 Fundamental principles of counting . . . . . . . . . . . . . . . . . . . . . . . . . . . . 73
4.1.3 Warm-up . . . . . . . . . . . . . . . . . . . . . . . . . . . . . . . . . . . . . . . . . . . 75
4.2 THE PRINCIPLE OF INDUCTION . . . . . . . . . . . . . . . . . . . . . . . . . . . . . . . . 77
4.2.1 Warm-up . . . . . . . . . . . . . . . . . . . . . . . . . . . . . . . . . . . . . . . . . . . 78
4.3 SEQUENCES AND SERIES . . . . . . . . . . . . . . . . . . . . . . . . . . . . . . . . . . . . 79
4.3.1 Sequences . . . . . . . . . . . . . . . . . . . . . . . . . . . . . . . . . . . . . . . . . . . 79
4.3.2 Check understanding . . . . . . . . . . . . . . . . . . . . . . . . . . . . . . . . . . . . . 79
4.3.3 Fibonacci sequence . . . . . . . . . . . . . . . . . . . . . . . . . . . . . . . . . . . . . . 80
4.4 PIGEON HOLE PRINCIPLE . . . . . . . . . . . . . . . . . . . . . . . . . . . . . . . . . . . . 80
4.4.1 Definition . . . . . . . . . . . . . . . . . . . . . . . . . . . . . . . . . . . . . . . . . . . 80
4.4.2 Examples . . . . . . . . . . . . . . . . . . . . . . . . . . . . . . . . . . . . . . . . . . . 81
CONTENTS 7

4.4.3 Exercises . . . . . . . . . . . . . . . . . . . . . . . . . . . . . . . . . . . . . . . . . . . 81
4.5 GAME THEORY . . . . . . . . . . . . . . . . . . . . . . . . . . . . . . . . . . . . . . . . . . . 82
4.5.1 WINNING STRATEGY . . . . . . . . . . . . . . . . . . . . . . . . . . . . . . . . . . . 82
4.5.2 Examples . . . . . . . . . . . . . . . . . . . . . . . . . . . . . . . . . . . . . . . . . . . 82
4.5.3 Exercises . . . . . . . . . . . . . . . . . . . . . . . . . . . . . . . . . . . . . . . . . . . 83
4.6 PROBLEMS . . . . . . . . . . . . . . . . . . . . . . . . . . . . . . . . . . . . . . . . . . . . . 84

II RWANDA MATH OLYMPIAD GLOSSARY 89


8 CONTENTS
Part I

MATH OLYMPIAD CONTENT

9
Chapter 1

ALGEBRA

1.1 SETS AND SUBSETS


1.1.1 History of numbers
The Babylonians developed a place-value system based on the numerals 1 (one) and 10 (ten). The ancient
Egyptians added to this system to include all the powers of 10 up to one million.There is some archaeo-
logical evidence that suggests that humans were counting as far back as 50,000 years ago in South Africa.
The following order of subsets of numbers ranges from Natural numbers to Complex numbers.

Natural numbers(N)
People started doing math by counting things and they were able to find solutions to equations like x + 3 = 5
in Positive Integers. Eg: 1,2,3,4,..... This set of Natural numbers is the mother of bigger sets of numbers we
have in mathematics.

Integer numbers(Z)
Everything was okay with Positive integers until people realized they can’t get solutions to equations like
x + 5 = 3 so they had to invent Negative numbers which in union with natural numbers make the whole set
of Integers.
Eg: -2, -1,0,1,2.

Rational numbers(Q)
There was a quest to find numbers that could satisfy equations like 7x = 1 and this led to the invention of
numbers that can be written as the division of two integers.
Eg:5 = 15 = 15 3 1
3 , 1.5 = 2 and 0.3333333 = 3

Irrational numbers(I)
Some√equations like x2 = 2 led to the invention of numbers that can’t be written as division of integers.
Eg: 2, π, e, ln2 ... i.e: They are Irrational.

Real numbers(R) This is the union of Rational and Irrational numbers.

Imaginary numbers √
There was no need to invent other numbers till the 17th century René Descartes said: let −1 = i and
this was useful in many areas ranging from electricity to Albert Einstein’s theories. Now we are able to find
solutions to equations like x2 = −1 , log(−1) .

Complex numbers(C)
This is the union of Real numbers and Imaginary numbers. Eg: 2 + 5i

11
12 CHAPTER 1. ALGEBRA

x
Fun fact: It seems like we know all numbers that do exist in the universe, but what is 0 ?!!!! Do we need
another invention?

1.1.2 Sets and subsets


Sets
Examples of sets: Real numbers(R), Rational numbers(Q), Integers(Z), Whole numbers,among others. [1]

Subsets
A subset is a set whose elements are all members of another set. A set A is a subset of another set B if all
elements of set A are elements of set B.
The symbol ⊆ means ”is a subset of”. The symbol ”⊂” means ”is a proper subset of”. Since all of the
members of set A are members of set D, A is a subset of D. Symbolically this is represented as A ⊆ D.
A proper subset is one that contains a few elements of the original set whereas an improper subset contains
every element of the original set along with the null set.
A subset which contains all the elements of the original set is called an improper subset. It is denoted by ⊆.

N.B: If a set has “n” elements, then the number of subsets of the given set is 2n and the num-
ber of proper subsets of the given subset is given by 2n − 1. [1]
EXAMPLES:
Subsets of Integers(Z+, Z−, even numbers, odd numbers, prime numbers, etc).

Number theory is one of the 4 branches of Olympiad mathematics that deals with integers.
The following are some common subsets of integers:

• Even numbers: Those are integers that are divisible by 2 they can be expressed as 2n ∀ n ∈ Z
Example: Prove that 4n²+6n+8 is always even ∀ n ∈ Z
Solution: 4n² +6n+8 can be written as 2(2n²+3n+4) which is a multiple of 2. Hence proven.

• Odd numbers: Those are integers that are not divisible by 2 they can be expressed as 2n + 1 ∀ n ∈
Z

Example: Prove that (2n + 1)2 is always odd ∀ n ∈ Z


Solution: (2n + 1)2 can be expanded to be 4n2 + 4n + 1 which equals 2(2n2 + 2n) + 1 which is an
even number plus 1 hence completing the proof.

• Prime numbers: Those are natural numbers that have only 2 divisors, that is 1 and itself. Eg:
2,3,5,7,11,13,... There are infinite prime numbers (Hippasus the disciple of Pythagoras proved this by
showing its opposite is wrong)

Example: Proof:
A prime number can either be 2 or greater than 2, so we divide it into cases.
Case 1: n = 2
When n = 2, n + 1 = 3, which is divisible by 3.
Case 2: n > 2 and n is odd
Since all primes greater than 2 are odd, if n is odd, then n + 1 is even. An even number is divisible by
2.
Thus, in both cases, n + 1 is either divisible by 2 or divisible by 3 when n is a prime number.

• Square numbers (perfect squares): These are integers that also have integer square roots e.g., 1,
4, 9, 16, ....
1.1. SETS AND SUBSETS 13

There are many properties these kinds of integers possess concerning divisibility. For example, n2 −
m2 = (n − m)(n + m).
Example: Prove that the square of an even number is divisible by 4.
Solution: Let’s say that the even number is 2n for some integer n. Then its square will be 4n2 , which
is a multiple of 4, providing a proof.

Subsets of Real numbers

Algebra in Olympiad Mathematics and Real Numbers

Algebra is a branch of Olympiad mathematics that deals especially with manipulating and solving
mathematical expressions. One of the fundamental sets of numbers in algebra is the set of Real Numbers,
denoted by R.

The set of Real Numbers is the union of Rational Numbers (Q) and Irrational Numbers (I). Rational
Numbers are numbers that can be expressed as a ratio of two integers, whereas Irrational Numbers cannot
be expressed as such. The set of Real Numbers, R, includes both the rational and irrational numbers.

• Rational Numbers (Q) - These are numbers that can be expressed as a ratio of two integers, where the
denominator is not zero. Examples of rational numbers include 12 , 34 , −2, 0, etc.

• Irrational Numbers (I) - These are numbers that cannot be expressed as a simple fraction and have

non-terminating, non-repeating decimal expansions. Examples of irrational numbers include π, 2,

3, etc.

• Integer Numbers (Z) - These are the set of whole numbers, including their negatives and zero. It
consists of all positive and negative integers as well as zero. Examples include −3, −2, −1, 0, 1, 2, 3,
etc.

• Natural Numbers (N) - These are the set of positive integers, starting from 1 and counting upwards
without end. Examples include 1, 2, 3, 4, etc.

The set of Real Numbers, R, is a very large and diverse set, encompassing all possible numbers on the
number line.

N.B: The above diagram shows us the Real numbers (as a union of Rational numbers and Irrational num-
bers.) Note that, from the diagram, you can easily see the other subsets of Rational numbers which are
integers, whole numbers, and natural numbers.

EXERCISES
[2]
1. State True or False:
14 CHAPTER 1. ALGEBRA

a. All whole numbers are natural numbers


b. All whole numbers are natural numbers
c. All rational numbers are integers.
d. Fractions are non-integers.
2. How many subsets does the set A = {1, 2, 3, 4, 5} have?

3.If U = {1, 3, 5, 7, 9, 11, 13}, then which of the following are subsets of U
a. B = {2, 4} b, A = {0}
b. C = {1, 9, 5, 13}
c. D = {5, 11, }
d. E= {13, 7, 9, 11, 5, 3, 1}
e. F = {2, 3, 4, 5}
4.Which of the following sets is a universal set for the other four sets?

a. The set of even natural numbers


b. The set of odd natural numbers
c. The set of natural numbers
d. The set of negative numbers
e. The set of integers

5.Write all the subsets for the following


a. {0}
b. {6, 11}
c. {2, 5, 9}
d. {1, 2, 6, 7}
e. {a, b, c}
f. ∅
g. {p, q, r, s}
6.Write down all the possible proper subsets for each of the following
a. {a, b, c, d}
b. {1, 2, 3}
c. {p, q, r}
d. {5, 10}
e. {x}
f. ∅
7. Find the number of subsets for set.
1.1. SETS AND SUBSETS 15

a. containing 36 elements
b. whose cardinal number is 5
8.Find the number of proper subsets for set
a. containing 13 elements
b. whose cardinal number is 17
9.Show with an example that if the number of elements in a set is ‘n’ , then
a. he number of subsets is 2n
b. the number of proper subsets is 2n − 1
10. Let A = {2, 3, 4, 5, 6, 7} ,B = {2, 4, 7, 8} , C = {2, 4}. Fill in the blanks by ⊂ or ̸⊂to make the
resulting statements true.
a. B —– A
b. C —– A
c. B —– C
d. ∅ —– B
e. C —– C
f. C —– B
11.State whether true or false.
a. Quadrilateral ⊆ polygon
b. {1} ↔ {0}.
c. Whole numbers ⊆ natural numbers
d. a ∈ d, e,f, a
e. Natural numbers ⊆ whole numbers
f. Integers ⊆ natural numbers
g. 0 ∈ ∅
h. ∅ ∈ {1 , 2, 3 }
12. Let Ax : x = n − 2, n < 5. Find A when
a. n = W, n ∈ W
b. n = N, n ∈ N
c. n ∈ I = I
2
13. Let 7 and 4 be two rational numbers. Show and explain why their product and division is also rational.

14. Let 4 and 4 are rational and irrational numbers. Show and explain why their addition and sub-
traction results in an irrational number.

15. f x + 4 = −11 + 2(x + 3). In the equation shown, f is a constant. For what value of f does the
equation have no real solutions?

16. 4 − 3y = 6y + 4 − 9y . Which of the following best describes the solution set to the equation shown?
16 CHAPTER 1. ALGEBRA

a. The equation has no solutions

b. The equation has exactly one solution, y = 0.

c. The equation has exactly one solution, y = 43 .

d. The equation has infinitely many solutions



17. If a = 5 + 2 6 and b = a1 , what will be the value of a2 + b2 ?

18. If a = 2 + 3, find the value of a − ( a1 )

19. Show that 5 + 3 is irrational.

20. What is the number of subsets of the set {1, 2, 3, 4, 5, 6, 7, 8, 9 , 10} having 3 elements?

1.2 FUNCTIONS
1.2.1 Input & Output, Domain, Co-domain and Range, Injective, Surjective
and Bijective Functions
1.2.2 Input & Output
An exchange rate calculator inputs a value in USD (1 dollar) and outputs the equivalent value in Rwandan
Francs (e.g. 970 RWF). What happens if it inputs 5 dollars instead? Can you describe the function that it
is applying?

The above function f (x) = 970x is an example of a function because it takes an input and maps to an
output. It is always the same output for a given input. (A different function would be used on a different
day with a different exchange rate).

We could also define the function to have two inputs: the number of dollars to change and the exchange rate
which will change every day. Then the function would be f (r, x) = rx where r is the exchange rate and x is
the number of dollars.

You should think of a function as a machine that takes some input(s) and produces some out-put. The
key thing to remember is that if you give a function the same input multiple times, it will always give the
same output. It cannot have multiple possible outputs for an input.

Therefore, a function can be defined as a relation between a set of inputs having at most one out-put
each.

1.2.3 Domain, Co-domain and Range


A function is only ever considered fully-defined if it has a domain and a co-domain.
Both of these are generally written in set terminology so it can be R (the real numbers), Q (the rational
numbers), Z (the integers), N (the natural numbers) or even just an arbitrary set (e.g. 0,1). The set can
even be non-numeric such as A,B,C,D or pairs (Z×Z) which would be a pair of integers like (1,-1).

Domain: The domain is the set of values that the input can take.

Co-domain: The co-domain is a set of all possible values (outputs) which can come out as a result.

Range or Image: The range is the set of actual outputs of a function. i.e. it is the set of values which
actually comes out.
1.2. FUNCTIONS 17

Examples

1. In the exchange rate function calculator f (x) = 970x above, we would say that the domain is the positive
real numbers (R+) as any real number can be converted but you can’t exchange negative dollars and the
co-domain would also be the positive real numbers. So a full function definition would be f : R+→ R+
where f (x) = 970x.

2. Consider the function f : R → R such that f (x) = 0. The domain of this function is all reals and the
co-domain which is all allowed values of the output are reals. But actually, only one value ever comes out
which is 0. So the range or image is just {0}.

In fact, the range is always a subset of the co-domain. In fact, the range is always a subset of the
co-domain but it is precisely those that are actually taken on while the co-domain can be much larger.

3. In the function f (r, x) = rx, we would say that f:R+→ R+ to say that the domain is the set of pairs
of positive reals (the exchange rate and the amount being converted) and the co-domain is the set of positive
reals (the amount in the new currency).

1.2.4 EXERCISES
1. What is the domain, co-domain and range of : R+→ R+ where f(x) = 1?

2. What is the domain, co-domain and range of : R+→ R+ where f (x) = 2x?

3. What is the domain, co-domain and range of : R → R where f (x) = x2 ?

4. What is the domain, co-domain and range of : Z → R where f (x) = x?

5. What is the domain, co-domain and range of: R × R → R where f (x, y) = x + y?

6. What is the domain, co-domain and range of : Q → Z where f (x) = 0 if x is positive and f (x) = 1 if
x is negative?

7. What is the domain, co-domain and range of : Z → Z where f (x) is the number of positive factors
that x has.

8. What is the domain, co-domain and range of : X → Z where X is the set of all possible English words
less than 8 letters long and f (x) is the length of the word x

1.2.5 Injective, Surjective and Bijective Functions


There are 3 special types of functions that are important.

Injectivity

[3]
I. A function is defined to be injective or one-to-one if there are no two elements in the domain that map
to the same element in the co-domain.
One-to-One or Injective functions define that each element of one set, say Set (A) is mapped with a
unique element of another set, say, Set (B).
18 CHAPTER 1. ALGEBRA

Example:
a, If function f: R→ R, then f (x) = 2x is injective. b, If function f: R→ R, then f (x) = 2x + 1 is in-
jective. c, If function f: R→ R, then f (x) = x2 is not an injective function, because here ifx = −1, then
f (−1) = 1 = f (1). Hence, the element of co-domain is not discrete here.

Surjectivity
II. A function f : A → B is surjective or onto if the range (image) is the same as the co-domain, meaning
that every element in the co-domain B has a corresponding value in the domain A that maps to it.

In the first figure, you can see that for each element of B, there is a pre-image or a matching element
in Set A. Therefore, it is an onto function. But if you see in the second figuren one element in Set B is
not mapped with any element of set A, so it’s not an onto or surjective function.

Examples:
1, Given that the set A = 1, 2, 3, set B = 4, 5 and let the function f = (1, 4), (2, 5), (3, 5). Show that
the function f is a surjective function from A to B. Given that the set A = {1, 2, 3}, set B = {4, 5}, and the
function f = {(1, 4), (2, 5), (3, 5)}, we can show that f is a surjective function from A to B:
Solution: Domain: A = {1, 2, 3} We can see that the element from set A, 1, has an image 4, and both 2
and 3 have the same image 5. Thus, the range of the function is {4, 5}, which is equal to set B. Therefore,
we conclude that f : A → B is an onto function. Hence, the given function f is a surjective function.

2, Prove if the function g : R → R defined by g(x) = x2 is a surjective function or not:


1.2. FUNCTIONS 19

Solution:
For the given function g(x) = x2 , the domain is the set of all real numbers, and the range is only the set
of square numbers, which does not include all the set of real numbers. Hence, the given function g is not a
surjective function.

Bijectivity

III. A function is defined to be bijective if it is both injective and surjective.

Examples:

1. For A = {−1, 2, 3} and B = {1, 4, 9}, f : A → B defined as f (x) = x2 is bijective.

2. Show that the one-to-one function f : {1, 2, 3} → {4, 5, 6} is a bijective function:


Solution: The given function f : {1, 2, 3} → {4, 5, 6} is a one-to-one function, and hence it relates every
element in the domain to a distinct element in the co-domain set. The three elements of the domain set
relate to all the three elements of the co-domain set. Also, since the co-domain includes all the elements of
the second set, the given function is also an onto function as the range is equal to the co-domain. Therefore,
the given function is a bijective function.

Summary:

If there is an injective function from domain X to co-domain Y , then we know that each element in X maps
to a unique element in Y , and so Y must be at least as big as X. If there is a surjective function from
domain X to co-domain Y , then we know that every element in the co-domain has at least one corresponding
element in the domain, which means that Y must be at least as big as X. If a function is bijective, then
both of those things must be true, and so the sets X and Y are the same size. This is one way (very useful
in combinatorics) of showing that two sets are the same size - by constructing such a function.

1.2.6 EXERCISES
Determine the nature of the following functions:
1. f : R → R where f (x) = x

2. f : Z → R where f (x) = x

3. f : R → Z where f (x) = x

4. f : R → R where f (x) = 2x

5. f : R → R where f (x) = 2x

6. f : R → R where f (x) = 0

7. f : R → R+ where f (x) = x2

8. f : R → R+ where f (x) = x

9. f : R+ → R+ where f (x) = x

10. f : R × R → R where f (x) = x + y


20 CHAPTER 1. ALGEBRA

1.2.7 Linear Functions


Introduction
[4] Linear functions are represented in the form y = f (x) = ax + b. These functions correspond to straight
lines on the Cartesian plane. The independent variable is x and the dependent variable is y. The slope of
the linear function is denoted by a, and the constant term or the y-intercept is represented by b.
As a linear function represents a straight line on the Cartesian plane, its equation can be used to find the
slope and y-intercept, which provide valuable information about the line’s characteristics and behavior.
Please note that linear functions have one independent variable and one dependent variable and can be
expressed in the form y = f (x) = ax + b.

A linear function has one independent variable and one dependent variable. The independent variable
is x and the dependent variable is y. The slope of the linear function is “a”. The constant term or the
y-intercept is “b”.

As a linear function represents a straight line on the graph, this means that when we have a linear function
then we also have an equation of the line.

Forms of equations of a line

1.2.8 Common terms associated with linear functions.


Slope/Gradient
The slope or gradient of a line represents the change in the y-coordinate with respect to the change in the
x-coordinate. It is denoted by ”m” and can be calculated as follows:

∆y
m=
∆x
Where: ∆y is the change in the y-coordinate, and ∆x is the change in the x-coordinate.
The slope formula between two points P(x1, y1) and Q(x2, y2) on a straight line is given by:
y2 − y1
m=
x2 − x1
The equation for the slope of a line passing through the point (x1, y1) is also known as the point-slope
form of the equation of a straight line and is given by:

y − y1 = m(x − x1 )
Here, ”m” represents the slope and (x1, y1) is a point on the line.
The slope-intercept form (general form) of the equation of a line is given by:
1.2. FUNCTIONS 21

y = mx + b
Where ”m” is the slope, and ”b” is the y-intercept.
Note:

1.Slope of Vertical Lines:


Vertical lines have no slope, as they do not have any steepness. In other words, we cannot define the steep-
ness of vertical lines. The slope formula,
y2 − y1
m=
x2 − x1
,
becomes undefined for vertical lines because both x2 and x1 have the same value, resulting in a division by
zero. Therefore, the slope of vertical lines is undefined.

2. Slope of Horizontal Lines: The slope of a horizontal line is equal to 0, as the y-coordinates of
all points on the line are the same. When using the slope formula
y2 − y1
m=
x2 − x1
, the numerator becomes zero since y2 and y1 are equal. Consequently, the slope of horizontal lines is 0.

3. Slope for parallel lines:


For two parallel lines given by l1 and l2 with the slopes m1 and m2 respectively, their slopes must also be
equal, i.e., m1 = m2 .

4. Slope for perpendicular lines:


For two lines l1 and l2 to be perpendicular, the product of their slopes must be equal to −1. Thus,
m1 × m2 = −1.

Examples:
1. Find the slope of a line between P (−2, 3) and Q(0, −1). Solution: Given, P (−2, 3) and Q(0, −1) are the
two points. Hence, the slope of the line,
−1 − 3 −4
m= = = −2
0 − (−2) 2

2. The line y = 3x + 1 is perpendicular to the other line passing through (3, 4). Find the slope and
equation of that other line.

Solution:
As the line y = 3x + 1 is perpendicular to the line passing through (3, 4), then m1 × m2 = −1.
As m1 = 3, then m2 = − 31 .Thus, the slope of the other line passing through (3, 4) is − 13 . The equation of
the other line passing through (3, 4) is
1
(y − 4) = − (x − 3)
3
or
1
y =− x+5
3

The x-intercept and y-intercept.


Intercept: The point where the line or curve crosses the axis of the graph. The meaning of intercept of
a line is the point at which it intersects either the x-axis or y-axis. If the axis is not specified, usually the
22 CHAPTER 1. ALGEBRA

y-axis is considered. It is normally denoted by the letter ‘b’.

If a point crosses the x-axis, then it is called the x-intercept. If a point crosses the y-axis, then it is
called the y-intercept.

Practice Examples.

1. Find the x-intercept and y-intercept for the line 5x − 8y = 2.

2. If the y-intercept of a line is -4 and the slope is 32 , then write its equation.

1
3. What is the equation of a line whose x and y-intercepts are given as 3 and -3?

Intersection of lines.
Point of intersection means the point at which two or more lines intersect. By solving the two equations of
lines simultaneously, we can then find the solution for the point of intersection of two lines. We can find the
point of intersection of three or more lines also.

For example, find out the point of intersection of two lines x + 2y + 1 = 0 and 2x + 3y + 5 = 0.

Solution:
Given straight line equations are: x + 2y + 1 = 0 and 2x + 3y + 5 = 0.
After solving the two equations simultaneously, we then get the intersection point which is (x, y) = (−7, 3).

EXERCISES
1. If the line that passes through the points (2,7) and (a,3a) has a slope of 2, the value of a is
5
a. 2

b. 10

c. 3
11
d. 5
12
e. 5

2. If x=3. Which of the following is true?

a. 2x = 5

b. 3x − 1 = 8

c. x + 5 = 3

d. 7 − x = 2

e. 6 + 2x = 14

3. What is the intersection point of these two lines, y = 2x + 6 and y = x + 4.

4. The list 11, 20, 31, 51, 82 is an example of an increasing list of five positive integers in which the
first and second integers add to the third, the second and third add to the fourth, and the third and fourth
add to the fifth.
How many such lists of five positive integers have 124 as the fifth integer?.

a. 10
1.2. FUNCTIONS 23

b. 7

c. 9

d. 6

e. 8

5. An expression that produces the values in the second row of the table shown, given the values of n in
the first row, is
n 1 2 3 4 5
value 1 3 5 7 9

a. 3n − 2

b. 2(n − 1)

c. n + 4

d. 2n

e. 2n − 1

6.Which equation represents the relationship between the values of x and y in the table

n 1.5 3 3 4
value 1 3 5 7

a. y = x + 0.5

b. y = 1.5x

c. y = 0.5x + 1

d. y = 2x − 0.5

e. x2 + 0.5

7. For what value of k is the line through the points (3, 2k + 1) and (8, 4k − 5) parallel to the x-axis?

a. -1

b. 3

c. 2

d. 0

e. -4

8. The y-intercepts of the three parallel lines are 2, 3, and 4. The sum of the x-intercepts of the three
lines is 36. What is the slope of these parallel lines?
1
a. 3

b. - 29

c. - 16

d. -4

e. - −1
4
24 CHAPTER 1. ALGEBRA

9. The line p is perpendicular to the line with equation y = x − 3. Line p has the same x-intercept as
the line with equation y = x − 3. The y-intercept of line p is

a. -3
1
b. 3

c. 3

d. -1

e. 0

10.In each row of the table, the sum of the first two numbers equals the third number. Also, in each
column of the table, the sum of the first two numbers equals the third number. What is the sum of the nine
numbers in the table?

m 4 m+4
8 n 8+n
m+8 4+n 6

a. 18

b. 42

c. -18

d. -6

e. 24

1.3 INEQUALITIES
1.3.1 Introduction
“Which is bigger? The sun or the moon”. We are always comparing
things in our daily life. When we look at inequalities, we are looking at
two expressions that are “inequal” or unequal to each other, as the name
suggests. In mathematics, Inequalities usually contain expressions
involving the symbols >, <, ≥ and ≤

1.3.2 Basic properties


For any real numbers a, b, c, d or (a, b, c, d ∈ R),

1. If If a ≥ b and b ≥ c, then a ≥ c. This follows directly from the


transitive property of inequalities. For example, If 8 ≥ 5 and 5 ≥ 3,
then 8 ≥ 3.

Transtivity: A relation between three elements such that if it


holds between the first and second and it also holds between the
second and third it must necessarily hold between the first and
third.

2. If a ≥ b and b ≥ a, then a = b. This is also a consequence of the


transitive property.
1.3. INEQUALITIES 25

3. If a ≥ b and c ≥ 0, then ac ≥ bc. This is because if a ≥ b and


c ≥ 0, then we can multiply both sides of the inequality a ≥ b by c
to get ac ≥ bc.

4. If a ≥ b and c ≤ 0, then ac ≤ bc. This is similar to the previous


property, but since c is negative, we have to flip the inequality sign.

5. If a ≥ b > 0, then 0 < a1 ≤ 1b . This property relates to the reciprocal


of the numbers a and b. If a ≥ b and are both positive, then we can
take the reciprocal of both sides to get a1 ≤ 1b . The inequality is
strict since a and b are not equal.

6. If a ≥ b and c ≥ d, then a + c ≥ b + d. This is a consequence of the


transitive property as well, since we can add b and d to both sides of
the inequality a ≥ b and c ≥ d to get a + c ≥ b + d.

These properties can be very useful when working with inequalities, and
they can be used to simplify expressions or to prove other inequalities.

1.3.3 Examples
1. By factorising, show that x2 + 2x + 1 ≥ 0 for all real x.

Solution: In other words, this question requires us to show why the


LHS(Left Hand Side): x2 + 2x + 1 can’t be negative.

By factorising,
x2 + 2x + 1

= x2 + x + x + 1

= x(x + 1) + x + 1

= (x + 1)(x + 1)

= (x + 1)2

Notice that, As it is stated in our question that x is a real number, then x + 1 is also a real number, hence
(x + 1)2 ≥ 0 This is because any square of a real number is always greater than or equal to
zero. This theorem is called the trivial inequality.

Trivial inequality: If x is a real number, then x2 ≥ 0 with x2 = 0 exactly when x = 0

2. For a triangle having sides 2x + 1, x + 1, and 4x − 7. Find the possible


values of x.

Solution: Notice that, it would be possible to find the value of x if and only if the question suggested
that we are using a right-angled triangle. Because, then the pythagoras theorem would help us. But the
question doesn’t specify which triangle to use, so we can’t use the pythagoras theorem.

Instead, the statement “the possible values of x” tells us that there are many possible values of x, hence
we are required to find all of them. To solve our question, we are going to use a theorem called the Triangle
inequality.
26 CHAPTER 1. ALGEBRA

Triangle inequality: If ∆ABC has side lengths BC = a, AC = b, and AB = c, then a + b > c and
a + c > b and b + c > a. In other words, the sum of any two sides of a triangle is greater than its third side.

By the triangle inequality then,

(2x + 1) + (x + 1) > (4x − 7),

(x + 1) + (4x − 7) > (2x + 1),

(4x − 7) + (2x + 1) > (x + 1).

After simplifying the above three inequalities, we then get x < 9, x > 37 , x > 15 . If we represent these three
inequalities on a number line, then we get to see that the possible values of x are in the range between 37
and 9, with 73 , 9 not included. In other words, 37 < x < 9.

3. Find all integer solutions to the inequality x2 − 4x − 3 < 0

Solution: Let’s try to bring the trivial inequality, and to do that we are going to make the LHS(Left
Hand Side) of our inequality a square. Hence,

x2 − 4x − 3 < 0

=⇒ x2 − 4x + 4 − 4 − 3 < 0

=⇒ (x − 2)2 − 7 < 0 As (x2 − 4x + 4) = (x − 2)2

=⇒ (x − 2)2 < 7

From the above inequality, (x − 2)2 < 7 but we know that by the trivial inequality (x − 2)2 ≥ 0. In other
words, it means (x − 2)2 can only be a positive number or zero, because a square of a number is always
a positive number or zero. So what numbers can the LHS be? Or, What integer squares are less than 7?
The integer squares that are less than 7 are (−2)2 , (−1)2 , 02 , 12 , 22 . This means that (x − 2) can be equal to
either −1, −2, 0, 1 or 2. Therefore, x can take the values 0, 1, 2 , 3 or 4 which are the integer solutions for
our inequality.

4. Let a, b and c be lengths of sides of a triangle and suppose that the


circumference of this triangle is 2. Show that

a2 + b2 + c2 + 2abc < 2.

Solution: The circumference or perimeter of ∆ABC is 2, =⇒ a + b + c = 2 Since a2 + b2 + c2 =


(a + b + c)2 − 2(ab + bc + ca) = 4 − 2(ab + bc + ca) as (a + b + c)2 = 22 = 4, hence the inequality can be
re-written as 4 − 2(ab + bc + ca) + 2abc < 2, i.e 2 − (ab + bc + ca) + abc < 1. If we substitute the 2 with a + b + c
we then get an equality (a + b + c) − (ab + bc + ca) + abc < 1,hence 1 − (a + b + c) + (ab + bc + ca) − abc > 0.
But 1 − (a + b + c) + (ab + bc + ca) − abc = (1 − a)(1 − b)(1 − c) and (1 − a)(1 − b)(1 − c) is necessarily > 0,
since each side of a triangle is shorter than the circumference.

Note that, we can also prove the above question by using the Triangle inequality. Try proving it on
your own by using the triangle inequality.
1.3. INEQUALITIES 27

1.3.4 EXERCISE A
[5]

1 □
1. What is the smallest integer that can be placed in the box so that 2 < 9

(A)7 (B)3 (C)4 (D)5 (E)6

2. Each of a, b, c and d is a positive integer and is greater than 3. If

1 1 1 1
= = =
a−2 b+2 c+1 d−3
then which of the following is true

(A)a < b < c < d (B)c < b < a < d (C)b < a < c < d
(D)d < a < c < b (E)b < c < a < d

3. If x is a number less than −2, which of the following expressions has the
least value?

(A)x (B)x + 2 (C) 12 x (D)x − 2 (E)2x

4. Kathy owns more cats than Alice and more dogs than Bruce. Alice owns
more dogs than Kathy and fewer cats than Bruce. Which of the
statements it must be true?

(A) Bruce owns the fewest cats (B) Bruce owns the most cats
(C)Kathy owns the most cats (D)Alice Kathy owns the most dogs
(E) Kathy owns the fewest dogs

5. In a group of five friends: Amy is taller than Carla. Dan is shorter than
Eric but taller than Bob. Eric is shorter than Carla. Who is the shortest?

(A)Amy (B)Bob (C)Carla (D)Dan (E)Eric

1 6 1
6. The number of integers n for which 7 ≤ n ≤ 4 is

(A)17 (B)18 (C)19 (D)20 (E)24

1.3.5 EXERCISE B
[5] [6]
1. By factorising, show that x2 − 8x + 20 ≥ 0 for all real x.
2. Show that x + x1 ≥ 2 for all real and positive x.
3. Determine if it is possible to form a triangle with the given side lengths. If not possible, explain why
not.

a. 3km, 4km, 5km

b. 18m, 20m, 40m

c. 9cm, 13m, 25cm

d. 9cm, 13m, 25cm


28 CHAPTER 1. ALGEBRA

4. For a triangle having sides 3x + 1, 2x + 1, and 5x − 6. Find the possible values of x.


5. Find all integer solutions to the inequality x2 − 8x − 6 < 0
6. If the measures of the sides of a right-angled√triangle are a,b and c, where
c is the hypotenuse. Show that a + b ≤ c 2

1.4 PROBLEMS
[5]
1. Barry has three sisters. The average age of the three sisters is 27. The average age of Barry and his
three sisters is 28. What is Barry’s age?

a. 1

b. 30

c. 4

d. 29

e. 31

2. Jack went running last Saturday morning. He ran the first 12 km at 12 km/h and the second 12 km
at 6 km/h. Jill ran the same route at a constant speed, and took the same length of time as Jack. Jill’s
speed in km/h was

a. 8

b. 9

c. 6

d. 12

e. 24

3. The regular price for a bicycle is $320. The bicycle is on sale for 20 % off. The regular price for a helmet
is $80. The helmet is on sale for 10 % off. If Sandra bought both items on sale, what is her percentage
savings on the total purchase?

a. 18%

b. 12%

c. 15%

d. 19%

e. 22.5%
1
4. When simplified, 2
2+ 3
is equal to

1
a. 8
5
b. 2
5
c. 8
1
d. 2
1.4. PROBLEMS 29

3
e. 8

5. The average of 1, 3 and x is 3. What is the value of x?


a. 4
b. 5
c. 2
d. 3
e. 1
6. There are two values k of for which the equation x2 + 2xk + 7k − 10 = 0has two equal real roots (that
is, has exactly one solution forx ). The sum of these values of k is
a. 0
b. 3
c. 3
d. -7
e. 7
7. If x = 2y and y ̸= 0 , then(x − y)(2x + y) equals
a. 5y2
b. y2
c. 3y2
d. 6y2
e. 4y2
8. If x = 18 is one of the solutions of the equation x2 + 12x + c = 0 , the other solution of this equation
is :
a. x = 216
b. x = −6
c. x = -30
d. x = 30
e. x = -540
a
9. The operation ⊗ is defined by a ⊗ b = b + ab . What is the value of 4 ⊗ 8?
1
a. 2

b. 1
5
c. 4

d. 2
5
e. 2
a b a−b
10. If b = 3 and c = 2 , then the value of c−b ?
30 CHAPTER 1. ALGEBRA

a. -4

b. - 14
2
c. 3

d. 2

e. 6
1 1 1 1
11. If a and b are positive integers such that a + 2a + 3a = b2 −2b , then the smallest possible value of
a+b

a. 8

b. 6

c. 96

d. 10

e. 50

12. If x and y are positive integers with x > y and x + xy = 391 , what is the value of x + y ?

a. 38

b. 39

c. 40

d. 41

e. 42

13. if x2 = 8x + y and y 2 = x + 8y with x ̸= y , then the value of x2 + y 2 is

a. 9

b. 49

c. 63

d. 21

e. 56
x−y x−z
14. If z−y = −10 then the value of y−z is

a. 11

b. -10

c. -9

d. 9

e. 10

15. For how many odd integers k between 0 and 100 does the equation
2 2
−n2 −4 2
+n2 +k
24m + 2m = 2k+4 + 23m

have exactly two pairs of positive (m, n) integers that are solutions?
1.4. PROBLEMS 31

a. 17
b. 20
c. 19
d. 18
e. 21
16. What is the sum of all numbersq which can be written in the form q = ab where aand b are positive

integers with b ≤ 10 and for which there are exactly 19 integers n that satisfy q < n < q ?
a. 871.5
b. 743.5
c. 777.5
d. 808.5
e. 1106.5
17. Three distinct integers a, b, and csatisfy the following three conditions:
• abc = 17, 995
• a, b and c form an arithmetic sequence in that order, and
• (3a + b), (3b + c), (3c + a) form a geometric sequence in that order
What is the value of a + b + c ?
(An arithmetic sequence is a sequence in which each term after the first is obtained from the previous term
by adding a constant. For example 3, 5, 7, is an arithmetic sequence with three terms.A geometric sequence
is a sequence in which each term after the first is obtained from the previous term by multiplying it by a
non-zero constant. For example, 3, 6, 12 is a geometric sequence with three terms.)
a. -63
b. -42
c. -68,229
d. -48
e. 81
18. Four different numbers a, b, c and d are chosen from the list −1, −2, −3, −4 and −5 . The largest possible
value for the expression ab + cd is
5
a. 4
7
b. 8
31
c. 32
10
d. 9
26
e. 25

19. Suppose that a, b, and c are integers with (x − a)(x − 6) + 3 = (x + b)(x + c) for all real numbers x. The
sum of all possible values of b is
a. -12
32 CHAPTER 1. ALGEBRA

b. -24
c. -14
d. -8
e. -16

20. If x and y are integers with (y − 1)x+y = 43 , then the number of possible values x is
a. 8
b. 3

c. 4
d. 5
e. 6
Chapter 2

NUMBER THEORY

2.1 INTEGERS: THE BASICS


2.1.1 Introduction
Integers are whole numbers that do not √ have any fractional or decimal parts.
√ Integers can be positive,
negative, or zero. For instance, 102 and - 4 are integers whereas 3.4, 12 or 2 are not integers. The set of
integers is denoted as Z and includes the following numbers: ...,-3,-2,-1,0,1,2,3,..

2.2 TYPES OF INTEGERS


• Positive integers are also called natural numbers or counting numbers. The set of natural numbers is
denoted as N or Z+ .
• Negative Integers are additive inverse of natural numbers. The set of negative natural numbers can
be denoted as Z− .
• Zero is neither positive nor negative. Zero has no sign.

N.B: Whole numbers are natural numbers or positive integers plus zero.

2.2.1 Fundamental arithmetic operations


The operations of integers refer to the basic arithmetic operations that can be performed with whole
numbers, including both positive and negative numbers. The four fundamental operations of integers are:
1. Addition (+)
As you learned in school, Addition is the process of combining two or more integers to find their total
or sum.
The rules of addition of integers are as follows,
When adding two positive integers, the result is a negative integer (e.g 5 + 3 = 8)
When adding two negative integers, the result is a negative integer (e.g (-7) + (-4) = -11)
When adding a positive integer to a negative integer, the result can vary (e.g 9 + (-2) = 7 and 5 +
(-20)= -15)
2. Subtraction (-)
Subtraction is the process of finding the difference between two integers.
To subtract an integer, you can add its additive inverse (the negative of the integer) instead.
Examples:

33
34 CHAPTER 2. NUMBER THEORY

9-4=5
(-3) - (-8) = 5
6 - (-2) = 8

3. Multiplication (×)
Multiplication is the process of repeated addition, to get a product.
Key takeaways:
When multiplying two integers with the same sign, the result is positive (e.g 3 × 4 = 12 and (-5) ×
(-2) = 10)
When multiplying two integers with different signs, the result is negative (e.g 6 × (-3) = -18)
Any integer multiplied by 0 is 0
A multiple is the product of that integer with any integer
For integers m and n, their product mn is both a multiple of m and n

4. Division (÷)
This operation divides one integer by another, producing a quotient.
m
We say that integer m is divisible y an integer n when n is an integer.
When you divide one integer by another, the result may or may not be an integer (e.g 6 ÷ 2 = 3
(integer result), 7 ÷ 2 = 3.5 (non-integer result))
Similar to multiplication, if both integers have the same sign, then the result is positive. If the
integers have different signs, then the result is negative.

2.2.2 Problems
Review problems
1. How many 4′ s must we add to get 44?

2. Is 22 a multiple of 4?

3. Is 63 a multiple of -9?

4. What is the value of (4 × 3) + 2?

5. Find the smallest positive integer that is both a multiple of 4 and 7.

6. Is zero a multiple of 5?

7. Is 100 divisible by 12?

8. Find all positive divisors of 28.

9. 28 less than five times a certain number is 232. What is the number?

Challenging problems
1. The values of r, s, t and u are 2, 3, 4, and 5, but not necessarily in that order. What is the largest
value of r × s + u × r + t × r?

2. Find the 5 largest negative multiples of 13.

3. Find the smallest natural number that is not a divisor of 5040.

4. Find the smallest integer that is not a divisor of 5040.

5. Is 1111 divisible by 41?


2.3. EVEN AND ODD NUMBERS 35

2.3 EVEN AND ODD NUMBERS


2.3.1 Definition
An even number is a number which has a remainder of zero upon division by 2, while an odd number is
a number which has a remainder of 1 upon division by 2.
If the units digit (or ones digit) is 1,3,5,7, or 9, the number is called an odd number, and if the units digit
is 0,2,4,6,or 8, the number is called an even number.
Thus, the set of integers can be partitioned into two sets based on parity:
• the set of even ( or parity 0) integers. Given n as an integer,every even number can be written as 2n
• the set of odd ( or parity 1) integers. Given n as an integer, every odd number can be written as
2n + 1
Parity is a fundamental property of integers, and many seemingly difficult problems can be solved easily
using parity arguments.
• Example 1: Figure out whether 1729 is an odd or even number.
Solution:
Since the remainder obtained on diving 1729 by 2 is 1, 1729 is an odd number OR the number 1729
is an odd number because it ends with digit ”9.”
• Example 2: Find out whether -1000 is an even or odd number.
Solution:
Since the remainder obtained when -1000 is divided by 2 is 0, -1000 is an even number OR -1000 is
an even number because it ends with digit ”0.”

2.3.2 Even and odd properties


The following are the parity properties of odd and even numbers:
• even ± even = even
• odd ± odd = even
• even ± odd = odd
• even × even = even
• odd × odd = odd
• even × odd = even

• Example 3: If n is an integer what is the parity of 2n + 2 (is 2n + 2 even or odd number?


Solution:
Since n is an integer, n + 1 is also an integer. Then, 2n + 2 = 2(n + 1) shows that the parity of 2n + 2
is 0, which implies 2n + 2 is always an even number.
• Example 4: Is the number (47630750675+453407032)×549068453 even or odd?
Solution:
Here, it would be unwise to actually multiply out these numbers. Instead, we can apply the
properties of even and odd numbers.
Since 47630750675 ends in a 5, it is odd. On the other hand, since 453407032 ends in a 2, it is even.
By property 3, even ± odd= odd, so 47630750675+453407032 is odd. Since that sum is being
multiplied by 549068453, which is odd, the entire number is odd since property 6 gives odd × odd=
odd.
36 CHAPTER 2. NUMBER THEORY

• Problem 5: If k is an integer, which of the following is always even: A) 2k + 1 B) k 2 C) 4k + 4 D)


K2 − 1
Solution:
A is always odd for any k B is odd whenever k is odd D is odd whenever k is even C can be written
as 4(k + 1), which means the remainder is 0 upon division by 2. The answer is C

2.3.3 Problems
Review problems
1. If a is a negative odd number and b is a positive even number, which of the following must be a
positive even number: a) b − a b) a + b c) ab d) −ab

2. Given that a and b are integers, the expression (a2 + a + 7) × (2b + 1) is: a) always odd b) always
even c) even or odd, depending on the values of a and b?

3. Let x and y be integers, which of the following is true about (x + y)2 + xy a) It is even b) It is odd c)
It is even if x is even d) It is even if y is even e) It is even if −xy is negative

4. Let P be the product of the first 100 prime numbers, What is the parity of P ?

5. If k is an integer, what is the parity of k 2 + k?

6. What is the sum of the first and the last even numbers between 1 and 100?

Challenging problems
1. Can an even number divided by another even number, times another even number ever equal to an
odd number? If ”yes,” find three numbers that work. If ”no,” then why not?
x2 +y 2 x+y
2. For integers x and y, show that 2 + 2 is an integer.

3. The product of the digits in 38 is even because 3×8=24. Similarly, the product of the digits in 57 is
odd because 5×7=35. How many 2-digit numbers have an odd product?

4. Prove that the sum of an odd number and an even number is odd.

5. Let n be an odd positive integer. Prove that the sum of three consecutive odd integers, beginning
with n, is always divisible by 3.

2.4 PRIMES and COMPOSITES


2.4.1 Introduction
Remember that 1 is a divisor of every natural number and that every natural number is a divisor of itself.
This means that every natural number greater than 1 has at least 2 positive divisors.
A prime number is a natural number greater than one that has no positive divisors other than 1 and
itself. For example, 5 is a prime number because it has no positive divisors other than 1 and 5.
The first 49 prime numbers are 2, 3, 5, 7, 11, 13, 17, 19, 23, 29, 31, 37, 41, 43, 47, 53, 59, 61, 67, 71, 73, 79,
83, 89, 97, 101, 103, 107, 109, 113, 127, 131, 137, 139, 149, 151, 157, 163, 167, 173, 179, 181, 191, 193, 197,
199, 211, 223, and 227.
In contrast to prime numbers, a composite number is a positive integer greater than 1 that has more
than two positive divisors. For example, 4 is a composite number because it has three positive divisors: 1,
2, and 4.
A composite number c can be written as a product of two integers both between 1 and itself:

c = ab
2.4. PRIMES AND COMPOSITES 37

where a an b are (not necessarily distinct) divisors of the composite number c.


N.B
• All positive integers greater than 1 are either prime or composite. 1 is the only positive integer that
is neither prime nor composite.
• 2 is the smallest and the only even prime number because all other even numbers are divisible by 2.
All other prime numbers are odd!

2.4.2 Identifying primes


The simplest way to determine whether a number is a prime is to use trial division. It involves testing
whether the number is divisible by any integer from 2 up to the square root of the number. If it is not
divisible by any of these integers, then it is a prime number. √
i.e If n is a composite number, then it must be divisible by a prime p such that p ≤ n.

• Example 1: Is 211 a prime number?


solution:√ If 211√is a prime number, then it must not be divisible by a prime that is less than √ or
equal to 211. 211 is between 14 and 15, so the largest prime number that is less than 211 is 13.
It is therefore sufficient to test 2, 3, 5, 7, 11, and 13 for divisibility. 211 is not divisible by any of
those numbers, so it must be prime.
• Example 2: What is the sum of the two largest two-digit prime numbers?
Solution: If a two-digit
√ number is composite, then it must be divisible by a prime number that is
less than or equal to 100 =10. Therefore, it is sufficient to test 2, 3, 5, and 7 for divisibility.
Counting backward,
99 is divisible by 3;
98 is divisible by 2;
97 is not divisible by 2, 3, 5, or 7, implying it is the largest two-digit prime number;
96 is divisible by 2;
95 is divisible by 5;
94 is divisible by 2;
93 is divisible by 3;
92 is divisible by 2;
91 is divisible by 7;
90 is divisible by 2;
89 is not divisible by 2, 3, 5, or 7, implying it is the second largest two-digit prime number.
The sum of the two largest two-digit prime numbers is 97+89=186
• Example 3: What prime number follows 1997/
Solution: 1998 is divisible by √ 2. If 1999
√ is composite, then it must be divisible by a prime number
that is less than or equal to 1999. 1999 is between 44 and 45, so the possible prime numbers to
test are 2, 3, 5, 7, 11, 13, 17, 19, 23, 29, 31, 37, 41, and 43. 1999 is not divisible by any of those
numbers, so it is prime.

Sometimes, testing a number for primality does not involve exhaustively searching for prime factors, but
instead making some clever observation about the number that leads to a factorization. The next example
demonstrate this:
• Example 4: Is 12345 a prime number?
12345
Solution: As every number that ends in ”5” is divisible by 5, We have 5 = 2469. So 12345 is not
a prime number.
38 CHAPTER 2. NUMBER THEORY

2.4.3 Prime factorization


A prime factorization of a positive integer is that number expressed as a product of powers of prime
numbers.
The uniqueness of prime factorization is an incredibly important result, thus earning the name of
Fundamental theorem of arithmetic: ”Any integer greater than 1 is either a prime number, or can be
written as a unique product of prime numbers, up to the order of the factors.”
This statement implies that if a number is not prime, it has a prime number as its factor. For example, the
factors of 10 are 1,2,5, and 10, where 2 and 5 are both prime numbers.
”Up to the order of the factors” means that it does not matter the order in which the product of the prime
numbers is written.

• Example 5: Give the prime factorization of 48.


Solution: 48 is divisible by the prime numbers 2 and 3. The highest power of 2 that 48 is divisible
by is 16 = 24 . The highest power of 3 that 48 is divisible by is 3 = 31 . Thus, the prime factorization
of 48 is 48 = 24 × 31 No other positive integer has this prime factorization
• Example 6: What are the prime factors of 60?
Solution: The factors of 60 are 1, 2, 3, 4, 5, 6, 10, 12, 15, 20, 30, and 60. The prime factors are 2,3,
and 5.

Prime factorization is indeed useful for counting the number of divisors of a given number.
For any natural number n > 1, with prime factorization

n = pa1 1 pa2 2 pa3 3 ...pann

where p is a prime number and a is a positive integer, we can find number of divisors using the formula,

(a1 + 1)(a2 + 1)(a3 + 1)...(an + 1)

• Example 7: How many divisors does 60 have? z Solution: The prime factorization of 60 is
22 × 3 × 5. Therefore,60 has (2 + 1)(1 + 1)(1 + 1) = 3 × 2 × 2 = 12. These factors are 1, 2, 3, 4, 5, 6,
10, 12, 15, 20, 30, and 60.
• Example 8: If x, y,and z are three distinct prime numbers such that N = x × y × z, how many
positive divisors does N have excluding 1 and itself?
Solution: Since N = x × y × z, we can conclude that x, y,and z are factors of N . Since x, y,and z
are primes, we can not factor them to get any other number, so that gives us a total of 3 numbers.
But wait, we know that if x and y are factors of N x × y is also a factor of N . So a combination of
two factors out of the three factors is also a divisor of N In other words, we have x × y, x × z, and
y × z as factors of N , which are another 3 in addition to the 3 above.
Note that x × y × z is also a combination that is a factor of N , but it equals the number itself and is
therefore omitted.
So we have a total of 6 divisors, excluding 1 and the number itself

2.4.4 Problems
Review problems
1. What is the largest 3-digit prime number?
2. In the following sequence, how many prime numbers are present? 121,12321,1234321,123454321,. . .
3. What is the first year in twenty-first century that is a prime number?
4. Is 9409 a prime number?
2.5. MULTIPLES AND DIVISORS 39

5. How many prime numbers are multiples of 3?

6. How many prime numbers are multiples of 10?

7. What is the smallest prime factor of 6125?

8. If n has 15 factors ( 1 and 15 inclusive) and 2n has 20 factors, what is the number of factors of 4n?

9. What is the smallest positive number that has exactly 14 divisors?

10. They are 25 primes less than 100. i stheir sum even or odd?

Challenging problems
1. Find the smallest composite number that has no prime divisors less than 10.

2. A group of 25 pennies is arranged in 3 piles such that each pile contains a different prime number of
pennies. What is the greatest number of pennies possible in any of the pile?

3. Consider the expression 2n − 1 for n as an integer greater than 1. What are the least two values for
which the expression does not produce a prime number?

4. Let p be a prime number greater than 3. Prove that p2 − 1 is always divisible by 6.

5. What is the smallest prime divisor of 52 3 + 71 7?

2.5 MULTIPLES and DIVISORS


2.5.1 Greatest common divisor
The greatest common divisor(GCD), also called the highest common factor(HCF), of two
numbers is the largest number that divides them both.
For instance, the greatest common factor of 20 and 15 is 5, since 5 divides both 20 and 15 and no larger
number has this property.
The concept is easily extended to sets of more than two numbers: the GCD of a set of numbers is the
largest number dividing each of them.
The GCD is traditionally notated as gcd(a,b), or when the context is clear, simply (a,b)

2.5.2 Computing the greatest common divisor


• The GCD of several numbers may be computed by simply listing the factors of each number
and determining the largest common one. While in practice this is terribly inefficient, for
particularly small cases it is doable by hand.
The process may be split up using the method of factor pairs: once one determines a factor a of an
integer n, the quotient na is a factor of n as well. For instance, since 2 is a factor of 24, 24
2 = 12 is a
factor of 24 as well.

Example 1:

Find the greatest common divisor of 30, 36, and 24.


Solution: The divisors of each number are given by;

30: 1,2,3,5,6,10,15,30; 36: 1,2,3,4,6,9,12,18,36; 24: 1,2,4,6,12,24

The largest number that appears on every list is 6, so this is the greatest common divisor:
gcd(30,36,24)=6
40 CHAPTER 2. NUMBER THEORY

• A somewhat more efficient method is to first compute the prime factorization of each number in
the set. The resulting GCD is the product of the primes that appear in every factorization, to the
smallest exponent seen in the factorizations.

Example 2:

Compute gcd(4200,3780,3528)
Solution: We have

4200 = 23 · 3 · 52 · 7; 3780 = 22 · 33 · 5 · 7; 3528 = 23 · 32 · 7

Since 2 appears in each of these factorizations, it will appear in the GCD as well. It is taken to the
smallest power seen in the factorizations, which in this case is 2. So the GCD will contain 22 in its
factorization. Continuing along these lines, we obtain a GCD of 22 · 3 · 7
• The euclidean algorithms Because large numbers are difficult to work with by hand, there are a
number of algorithms used to simplify the problem down to a manageable level. Since the GCD has
the property that
gcd(a, b, c) = gcd(gcd(a, b), c)
the GCD can be calculated ”two at a time”, e.g. if we wanted to find the GCD of 20, 28, and 24, we
could first find the GCD of 20 and 28 (which is 4) and then the GCD of 4 and 24 (which is also 4).
As a result, almost all algorithms focus on the simplest case of determining the GCD of two numbers.
The euclidean algorithm is based on the following key observation: if d divides a and d divides b, then
d also divides a − b. This means that the GCD of a and b is the same as the GCD of a − b and b,
which is progress since this makes the numbers smaller.
As a result, we can repeat this process to form an algorithm:
1. If a = b, stop. The GCD of a and a is, of course, a. Otherwise, go to step 2.
2. If a > b, replace a by a − b, and go back to step 1.
3. If b > a, replace b by b − a, and go back step 1.

Example 3:

Determine the greatest common factor of 84 and 132


Solution: We follow our algorithm,
1. a = 84 and b = 132. Since b > a, we replace b by b − a = 132 − 84 = 48
2. a = 84 and b = 48. Since a > b, we replace a by a − b = 84 − 48 = 36
3. a = 36 and b = 48. Since b > a, we replace b by b − a = 48 − 36 = 12
4. a = 36 and b = 12. Since a > b, we replace a by a − b = 36 − 12 = 24
5. a = 24 and b = 12. Since a > b, we replace a by a − b = 24 − 12 = 12
6. a = 12 and b = 12. Since a = b, the GCD is 12.
You may have noticed, for instance, that once we get to 36 and 12 we can ”skip” the a = 24 step.
The Euclidean algorithm is especially useful by hand, since oftentimes the GCD will be become
”obvious” by inspection once the numbers get low enough (e.g. you might have noticed at the a = 36,
b = 12 step that the GCD is 12 since 36 = 12 · 3).
In practice this can be made rather efficient, as described below:
1. Let a = b and b = y
2. Given x and y, use division algorithm to write x = yq + r, 0 ≤ r < y
3. if r = 0, stop as y is the gcd(a, b)
4. if r ̸=, replace (x, y) by (y, r) and go to step 2.
2.5. MULTIPLES AND DIVISORS 41

Example 4:

What is gcd(16457,1638)?
Solution We apply the euclidean algorithm
16457= 1638×10+77
1638= 77×21+21
77=21×3+14
21=14×1+7
14=7×2+0
he process stops since we reached 0, we obtain
7 = gcd(7, 14) = gcd(14, 21) = gcd(21, 77) = gcd(77, 1638) = gcd(1638, 16457).

Example 5:

21n+4
Prove that 14n+3 is irreducible for every integer n.
Solution;
21n+4
14n+3 is irreducible if and only if the numerator and denominator have no common factor, i.e. their
greatest common divisor is 1. Applying the Euclidean algorithm,

21n + 4 = (14n + 3) × 1 + (7n + 1)

14n + 3 = (7n + 1) × 2 + 1

7n + 1 = (7n + 1) × 1 + 0

Hence gcd(21n + 4, 14n + 3) = 1, which shows that the fraction is irreducible.

Coprimes

When the only common positive divisor between two integers is 1, we say that those numbers are
relatively prime. A pair of relatively prime integers is also sometimes called coprime, which means the
same thing.
i.e if gcd(a, b) = 1, where a and b are positive integers, then a and b are coprime.
For instance, 8 and 13 are relatively prime.
Note that neither integers need to be prime in order for them to be relatively prime 8 and 15 are both not
prime, yet they are relatively prime.
*Properties of coprimes:

1. 1 is co-prime with every number.

2. Any two prime numbers are co-prime to each other.

3. Any two successive numbers/ integers are always co-prime.

4. The sum of any two co-prime numbers are always co-prime with their product: 2 and 3 are co-prime
and have 5 as their sum (2+3) and 6 as the product (2×3). Hence, 5 and 6 are co-prime to each other.

5. Two even numbers can never form a coprime pair as all the even numbers have a common factor as 2.

6. If two numbers have their unit digits as 0 and 5, then they are not coprime to each other. For
example 10 and 15 are not coprime since their HCF is 5 (or divisible by 5)
42 CHAPTER 2. NUMBER THEORY

Example 6:
How many positive integers less than 25 are coprime to 25?
Solution:
We need to determine the count of numbers that share no common factors (other than 1) with 25.
The factors of 25, other than 1 and 25, are 5 only since 25 = 55 . The numbers less than 25 that share
factors with 25 are the multiples of 5 (excluding 25 itself since we are looking for numbers less than 25).
These multiples are: 5, 10, 15, and 20.
Subtract the number of numbers that share factors from the total count of numbers less than 25. Total
numbers less than 25 = 1, 2, 3, ..., 24 (24 numbers in total)
Number of numbers less than 25 that share factors with 25 = 4 (5, 10, 15, 20)
Number of numbers less than 25 that are coprime to 25 = Total numbers less than 25 - Number of
numbers that share factors = 24 - 4 = 20

2.5.3 Lowest common multiple (LCM)


The lowest common multiple (LCM) of a finite set of non-zero integers is the smallest positive number that
is a multiple of each integer in the set. It is a fundamental concept in number theory, and is closely related
to the greatest common divisor.

Example 7:
Find the LCM of 30 and 36.
Solution:
Here is a list of the positive multiples of each number:
30 : 30, 60, 90, 120, 150, 180, . . . ;
36 : 36, 72, 108, 144, 180, . . .
The first number that appears on both lists is 180.

2.5.4 Computing lowest common multiple


Listing multiples is a very ineffective way of computing the LCM in general.
However, if the prime factorization of the numbers is known, then computing the least common multiple
is much simpler. The primes in the factorization of the LCM are the primes that appear in the
factorizations of at least one member of the list, and their exponent is the maximum of the exponents that
appear in the individual factorizations.

Example 7:
Compute the lcm(4200, 3580, 3528)
Solution:
We have

4200 = 23 · 3 · 52 · 7; 3780 = 22 · 33 · 5 · 7; 3528 = 23 · 32 · 7

The LCM can be read off from these factorizations by taking the maximum exponent for each prime:
23 · 33 · 52 · 72 = 264600
Generalizing this example, if the prime factorizations of mand n are

m = pa1 1 pa2 2 ...pakk

;
n = pb11 pb22 ...pbkk
where pi is distinct prime numbers and ai and bi are nonnegative integers then,
max(a1 ,b1 ) max(a2 ,b2 ) max(ak ,bk )
lcm(a, b) = p1 p2 ...pk
2.6. DIVISIBILITY 43

Example 8:
What is the smallest positive integer n for which lcm(n, 30) = 180?
Solution:
The prime factorization of 30 is 30 = 2 × 3 × 5 and the prime factorization of 180 is 180 = 22 × 32 × 5.
So n must include the factor of 22 and the factor of 32 . Thus the smallest n is 22 × 32 = 36.

2.5.5 Properties of LCM


• The LCM of a list of numbers divides any other common multiple.

• The LCM of a list of numbers can be computed two at a time, e.g. lcm(a, b, c) = lcm(lcm(a, b), c).

2.5.6 Relationship between GCD and LCM


Recall that the greatest common divisor of a set of integers is the largest (positive) number which is a
divisor of each integer in the set. The two concepts are intimately related; in particular, they satisfy the
following theorem:
gcd(a, b) × lcm(a, b) = a × b.

2.5.7 Problems
2.5.8 Review problems
1. Without using calculator what is the gcd of 2442 and 171171?
246
2. What is 642 in lowest terms?

3. what is the largest possible value of the greatest common divisor of numbers 5n + 6 and 8n + 7,
where n is an arbitrary positive integer?

4. Write any one coprime number to 216 and 215.

2.5.9 Challenging problems


1. What is the gcd(2015! + 1, 2016! + 1)?

2. How many ordered pairs of (a, b) such that 100 ≤ a, b ≤ 1000

3. Given that two integers a and b, such that 13gcd(a, b) = lcm(a, b) and that a + b = 2016, what are
the values of a and b?

4. the traffic lights at three different locations change every 48 seconds, 72 seconds, and 108 seconds,
respectively. If they change simultaneously at 9 am, when is the next time they change
simultaneously?

2.6 DIVISIBILITY
2.6.1 Divisibility rules
A divisibility rule is a heuristic for determining whether a positive integer can be evenly divided by
another (i.e. there is no remainder left over). For example, determining if a number is even is as simple as
checking to see if its last digit is 2, 4, 6, 8 or 0.
Multiple divisibility rules applied to the same number in this way can help quickly determine its prime
factorization without having to guess at its prime factors.
44 CHAPTER 2. NUMBER THEORY

Basic divisibility rules

A positive integer N is divisible by

• 2, if the last digit of N is 0, 2, 4, 6, or 8;

• 3, if the sum of digits of N is a multiple of 3;

• 4, if the last 2 digits of N are a multiple of 4;

• 5, if the last digit of N is 0 or 5;

• 6, if N is divisible by both 2 and 3;

• 7, if subtracting twice the last digit of N from the remaining digits gives a multiple of 7 (e.g. 658 is
divisible by 7 because 65 - 2 x 8 = 49, which is a multiple of 7);

• 8, if the last 3 digits of N are a multiple of 8;

• 9, if the sum of digits of N is a multiple of 9;

• 10, if the last digit of N is 0;

• 11, if the difference of the alternating sum of digits of N is a multiple of 11 (e.g. 2343 is divisible by
11 because 2 - 3 + 4 - 3 = 0, which is a multiple of 11);

• 12, if N is divisible by both 3 and 4.

Example 1:

Without performing actual division, show that the number below is an integer: 1481481468 12
Solution:
From the divisibility rules, we know that a number is divisible by 12 if it is divisible by both 3 and 4.
Therefore, we just need to check that 1,481,481,468 is divisible by 3 and 4.
By applying the divisibility text of 3, we get 1 + 4 + 8 + 1 + 4 + 8 + 1 + 4 + 6 + 8 = 45which is divisible by
3. Hence 1,481,481,468 is divisible by 3.
Applying the divisibility test for 4, we get that the last two digits, 68, is divisible by 4. Hence
1,481,481,468 is also divisible by 4.
Now, since we know that 1,481,481,468 is divisible by both 3 and 4, it is divisible by 12. Therefore,
1481481468
12 is divisible by 12.

Example 2:

Find all the possible values of a such that the number 98a6 is a multiple of 3.
solution:
From the rules of divisibility, the number 98a6 is a multiple of 3 if and only if the sum of its digits
9 + 8 + a + 6 = 23 + a is a multiple of 3. Since 0 ≤ a ≤ 9, this implies that a = 1, 4, 7 are all the possible
values.

N.B
y
It is also useful to note that the notation x|y is used to say x divides y i.e. the fraction x gives an integer.
For example, 3|6 is true, but 2|7 is false.
2.6. DIVISIBILITY 45

2.6.2 Remainders
An integer a is divisible by another integer b (or is a multiple of b) if a can be written as b times another
integer:
a × b = (integer)
For example, 14 = 7 × 2 = 2 × 7, so 14 is divisible by 2 and 7. However, dividing 14 by any other integer
greater than 1 does not produce an integer result. For example, 14 ÷ 4 leaves a remainder because 4 does
not go evenly into 14. The multiples of the divisor 4 are

4, 8, 12, 16, 20, . . .

, so 12 ÷ 4 = 3 is the largest number of fours that go into 14. However, since 12 < 14, we have 14 − 12 = 2
left over, which we call a ”remainder of 2.” We say that 14 ÷ 4 = 3R2, read as ”fourteen divided by four
equals three remainder 2.”
Note that the remainder will always be less than the divisor; in this case, dividing by 4 will always leave a
remainder that is either 0, 1, 2, or 3. To find the remainder of a number n upon division by a divisor d, we
first find the largest multiple of d that goes into n, and the remainder r is the amount left over:

n = kd + rwithr ≤ d

Example 3:
If n is an integer that leaves a remainder of 2 upon division by 6, what is the remainder of n upon division
by 3?
Solution:
Since n leaves a remainder of 2 upon division by 6, we have n = 6k + 2, where 6k is the largest multiple of
6 that goes into n Now, this can also be written as n = 3(2k) + 2, and since 2¡3, we have 3(2k) is the
largest multiple of 3 that goes into n. This shows that the remainder of n upon division by 3 is also 2.

2.6.3 Problems
Review problems
1. Without performing division, explain why the number 987654321 is a multiple of 9.

2. Without performing actual division, show that 87456399 is not divisible by 11.

3. If we know an number is a multiple of 5, how many possibilities are there for the last two digits?

4. If n is an integer that leaves a remainder of 4 upon division by 6, what is the remainder of n upon
division by 3.

5. How many two-digit integers leave a remainder of 2 when divided by 8?

Challenging problems

1. For what values of a and b is 12ab a multiple of 99?

2. Is 65973390 divisible by 210?

3. Show that if the last 3 digits of a number N are abc, N is a multiple of 8 if and only if 4a + 2b + c is a
multiple of 8

4. What is the largest possible remainder when two-digit number is divided by the sum of its digits?

5. prove that the product of three consecutive integers is always divisible by 6.


46 CHAPTER 2. NUMBER THEORY

2.7 PERFECT SQUARES and POWERS


2.7.1 introduction
A perfect square is an integer that can be expressed as the product of two equal integers. For example, 100
is a perfect square because it is equal to 10 × 10. If N is an integer, then N 2 is a perfect square. Because of
this definition, perfect squares are always non-negative.
Similarly, a perfect cube is an integer that can be expressed as the product of three equal integers. For
example, 27 is a perfect cube because it is equal to 3 × 3 × 3. Determining if a number is a perfect square,
cube, or higher power can be determined from the prime factorization of the number.

2.7.2 Perfect squares


Example 1:
Make a list of 10 perfect squares from the smallest.
Solution:
We have
(±0) = 0; (±1) = 1; (±2) = 4; (±3) = 9; (±4) = 16;
(±5) = 25; (±6) = 36; (±7) = 49; (±8) = 64; (±9) = 81;
Thus, the answer is 0,1,4,9,16,25,36,49,64,81.

2.7.3 Properties of perfect squares


1. perfect squares cannot have a units digit of 2, 3, or 7.
2. The square of an even number is even and the square of an odd number is odd.
3. All odd squares are of the form 4n + 1, hence all odd numbers of the form 4n + 3, where n is a
positive integer, are not perfect squares
4. All even numbers of the form 4n + 2, where n is a positive integer, are not perfect squares
5. All even squares are divisible by 4.
6. If p divides a2 , then p divides a as well. From this, we can say that a number is a perfect square if its
prime factorization contains all primes raised to some even power. i.e perfect squares are of the form
2ak
p2a 1 2a2
1 p2 . . . pk

where pi is distinct prime numbers and a1 is a positive integer.

2.7.4 Perfect powers


A perfect power is the more general form of squares and cubes. Specifically, it is any number that can be
written as the product of some non-negative integer multiplied by itself at least twice. In other words, it is
of the form nm for some integers n ≥ 0 and m > 1.
The set of perfect powers is the union of the sets of perfect squares, perfect cubes, perfect fourth powers,
and so on. The perfect powers less than or equal to 100 are 0,1,4,8,9,16,25,27,32,36,49,64,81,100.

• Example 2: Which the greatest of the following:

28 , 36 , 46 , 54 , 74 , 103

Solution:
We have,
2.7. PERFECT SQUARES AND POWERS 47

28 = 512, 36 , = 72946 = 4096, 54 = 625, 74 = 2401, 103 = 1000


Thus 46 is the greatest among these numbers. Do note that there are better ways to determine which
is the greatest or least given a set of numbers; the above powers can all be easily computed.

• Example 3: Find the number of digits in 647 given 67 = 279936 and 77 = 823543.
Since we are given 67 and 77 , we can easily see that the number of digits of any positive integer
between 60 and 70 is in fact 13. Here’s how: If n is greater than m, then n raised to k greater than m
raised to k, where n, m are real numbers and k is a positive integer.
Knowing this, we can find the number of digits in 647 :

607 = 67 × 10000000 = 2799360000000, 707 = 77 × 10000000 = 8235430000000

Since both 607 and 707 have an equal number of digits and 607 is less than 647 ,but 647 is less than
707 , then 647 must also have 13 digits.

• Example 4: Find the smallest positive integer n such that n3 can be written as the sum of three
consecutive positive integers.
Solution:
Let’s assume the four consecutive positive integers are k, k + 1, and k + 2. So, we have:
n3 = k + (k + 1) + (k + 2)
Now, let’s simplify the right-hand side: n3 = 3k + 3
Since n3 is a perfect cube, 3k + 3 must also be a perfect cube.
Now, let’s consider k as a positive integer. We want to find the smallest n, so we should look for the
smallest k.
The smallest value of k that makes 4k + 6 a perfect cube is k = 8. Substituting this value into the
equation: n3 = 3 × 8 + 3 = 27
Therefore, the smallest positive integer n is 3 (since 33 = 27).

2.7.5 Problems
Review problems
1. What is the positive number a in the following equation: 52 + 122 = a2

2. What are the perfect squares between 301 and 399?

3. Which of the following is not a perfect square?


a)125 b)144 c)441 d)225

4. Given that n is an integer, is n2 + 2n + 1 a perfect square?

5. What is the smallest n, such that 2n is a perfect square?

Challenging problems
1. In the following equation, a, b and c are all distinct positive integer: a2 + b2 = c2 . Determine the
smallest possible value of c.

2. Find the unit digit of 72 023.


48 CHAPTER 2. NUMBER THEORY

2.8 PROBLEMS
1. Which of the following is not a multiple of 3?
a) 0 b) 3 c) 13 d) 21

2. Determine which of the following is true.


a) 1260 is divisible by 4, 5, 6, and 7. b) 1260 is divisible by 42. c) 1260 is not divisible by 11. d) none
of the above.

3. Problem 3: Consider the expression 2n − 1 for n > 1. What are the least two values for which the
expression does not produce a prime number?
a) 3, 4 b) 2, 6 c) 4, 5 d) 4, 6

4. How many integers between 1 and 100 that are divisible by 3, 5, or 7?


a) 55 b) 50 c) 53 d) 54

5. Which of the following is not an even number?


a) 0 b) 37 c) -289756834 d) 589228650
3

6. What is the smallest composite number that has no prime divisors less than 10?
a) 100 b) 79 c) 22 d) 121

7. Given that 103041 is a perfect square, how many factors does 254016 have?
a) 120 b) 105 c) 504 d) 28

8. For what values of a and b is 12ab a multiple of 99?


a) 8, 7 b) 8, 9 c) 7, 9 d) 5, 6

9. Which of the following is the smallest prime number?


a) 0 b) 1 c) 2 d) 3

10. How many two-digit positive integers leave a remainder of 4 when divided by 8?
a) 10 b) 11 c) 12 d) 13

11. What is the last digit of 4925


a) 1 b) 9 c) 3 d) 7

12. If n is a positive integer and the gcd(14n + 3, 12n + 1) = 11, what is the possible of n?
a) 11 b) 10 c) 5 b) 14

13. What is the lowest common multiple of 17 and 12?


a) 0 b) 29 c) 68 d) 204

14. If n is an even number and S = 3n + 6, which of the following is false?


a) S is always divisible by 3. b) S is always divisible by 6. c) S is always divisible by 4. d) none of
the above.

15. What is the largest possible remainder when two-digit number is divided by the sum of its digits?
a) 6 b) 7 c) 8 d) 9

16. Given that two integers a and b, such that 13gcd(a, b) = lcm(a, b) and that a + b = 2016, what are
the values of a and b?
a) 144, 1872 b) 1, 2015 c) 169, 1847 d) 504, 1512

17. Which of the following is not a perfect power?


a) 0 b) 81 c) 128 d) 175

18. Evaluate 5 × (2 × 34 ) ÷ +7 − 8
a) 162 b) 154 c) 130 d) 125
2.8. PROBLEMS 49

19. The product of the digits in 38 is even because 3×8=24. Similarly, the product of the digits in 57 is
odd because 5×7=35. How many 2-digit numbers have an odd product?
a) 30 b) 35 c) 25 d) 20
20. What is the value of

gcd(1, 65) + gcd(2, 65) + gcd(3, 65) + · · · + gcd(64, 65) + gcd(65, 65)

a) 225 b) 145 c) 243 d) 130


50 CHAPTER 2. NUMBER THEORY
Chapter 3

GEOMETRY

3.1 ANGLES AND THEIR PROPERTIES


Def: An angle is the figure formed by two rays, called the sides of the angle and sharing a common endpoint,
called the vertex of the angle.

Complementary Angles
Complementary angles are angle pairs whose measures add up to one right angle ( 14 turn, 90°, or π
2 radians).
If the two complementary angles are adjacent their non-shared sides form a right angle.

Supplementary Angles
Two angles whose measures add up to a straight angle ( 12 turn, 180°, or pi or π radians) are called supple-
mentary angles. If the two supplementary angles are adjacent (i.e., have a common vertex and share just
one side), their non-shared sides form a straight line. Such angles are called a linear pair of angles.

Vertically Opposite Angles (VOA)


A pair of angles opposite to each other, formed by two intersecting straight lines that form an ‘X’-like shape,
are called vertical angles or opposite angles or vertically opposite angles. They are abbreviated as vert. opp.
∠s.T heyarealwaysequal.

Corresponding Angles Postulate or CA Postulate


If two parallel lines are cut by a transversal, then corresponding angles are congruent.

Alternate Interior Angles Theorem or AIA Theorem


If two parallel lines are cut by a transversal, then alternate interior angles are congruent to each other.

51
52 CHAPTER 3. GEOMETRY

3.1.1 Triangle and its properties


Triangles preliminaries
A triangle is a geometrical shape which has three sides and three angles. It can also be defined as a polygon
formed by three non-collinear points. A triangle is also a polygon which has three edges and three vertices.
Among the types of triangle which exist includes:

• Equilateral triangle. An equilateral is a triangle whose equal sides and length. The angle is always
60 degrees, the sides are all equal.

• Isosceles triangle. An isosceles triangle is the one which has two equal sides and angles. the sides b
are all equal and the angles are equal.

There are also other types of triangles which can be defined using the property of their angles. A triangle
which has one of its internal angles is exactly equal to 90 degrees “Right triangle”, while one with its
internal all are less than 90 is called an “Acute triangle” and a triangle whose one of the internal angles
is greater than 90 is called “Obtuse triangle.”

Similarities of triangles
Similarities refers to the property of two or more shapes having the same shape but not necessarily the same
size. When two shapes are similar, their corresponding angles are equal, but their corresponding sides are
proportional. Similar shapes can be obtained by uniformly scaling one shape to match the size of the other
shape. Similarity is denoted by the symbol ∼ . To prove the similarity of shapes, it is necessary to establish
that all corresponding angles are equal and the corresponding sides are in proportion. In geometry, two
triangles are considered similar if their corresponding angles are equal, and their corresponding sides are in
proportion. When triangles are similar, they have the same shape but may differ in size. This relationship
allows us to make comparisons and establish proportional relationships between the corresponding sides and
angles.

Key properties and concepts:


• Angle-Angle (AA) Similarity: If two angles of one triangle are congruent to two angles of another
triangle, then the triangles are similar. This property is known as the Angle-Angle Similarity Postulate.

• Side-Side-Side (SSS) Similarity: If the corresponding sides of two triangles are proportional, then
the triangles are similar. This property is known as the Side-Side-Side Similarity Theorem.

• Side-Angle-Side (SAS) Similarity: If one pair of corresponding sides of two triangles is propor-
tional, and the included angles are congruent, then the triangles are similar. This property is known
as the Side-Angle-Side Similarity Theorem.
3.1. ANGLES AND THEIR PROPERTIES 53

• Ratio of Corresponding Sides: In similar triangles, the ratios of corresponding sides are equal.
This property is known as the Triangle Proportionality Theorem. For example, if two triangles are
similar and their corresponding sides have lengths a and b, then the ratio of their side lengths is a/b.

Examples

1. In the triangle ABC, the point P lies closer to point A in the third of the line AB, the point R is
closer to point P in the third of the line P , and the point Q lies on the line BC so that the angles
∠P CB and ∠RQB are identical.
Determine the ratio of the area of triangles ABC and P QC.

Step-by-step explanation:
|AB| = a

1
|AP | = a
3

1 2 2
|P R| = · a= a
3 3 9
. Similarly,
 
1 2 4
|RB| = 1− − a= a
3 9 9

Area of triangle ABC is


1
S(ABC) = a·h
2
Area of triangle AP C is
1 1 1
S(AP C) = |AP | · h = a · h = · S(ABC)
2 3 3

Area of triangle P CB is
2
S(P CB) = S(ABC) − S(AP C) = · S(ABC)
3
54 CHAPTER 3. GEOMETRY

Let h2 be the height of triangle P CB. We have h2 = 23 h


Area of triangle P QB is
1 1 9 2 9
S(P QB) = |P B| · h2 = · a · h = · S(ABC)
2 2 4 3 4

Area of triangle P QC is
 
1 9
S(P QC) = S(ABC) − S(AP C) − S(P QB) = S(ABC) 1 − −
3 4

Let k be the ratio of the area of triangles ABC and P QC. We have
1 9 9
k =1− − = ≈ 0.2222
3 4 2

So,
S(ABC) 1 1 2
p= = = 9 = ≈ 0.2222
S(P QC) k 2
9

Therefore, p = 9 : 2
3.1. ANGLES AND THEIR PROPERTIES 55

Solved example. Take △ABC. If D ∈BC so that AD bisects ∠BAC, thenshowthatAB BD= AC
CD
Solution:
Consider this diagram below;

Then
∠ABC = ∠ECD
and
∠DEC = ∠DAC

so it follows that
△ABC ∼ △ECD
.

Thus,
AB EC
=
BD CD
.
Finally,
∠CED = ∠DAC
so it follows that △ACE is isosceles and AC = EC so we find that

AB AC
=
BD CD
as desired.

Congruence of triangles
Congruence refers to the property of two or more shapes having the same size and shape. When two shapes
are congruent, it means that they can be superimposed onto each other, preserving both their size and shape.
The corresponding sides and angles of congruent shapes are equal. Congruence is denoted by the symbol ∼ =.
To prove the congruence of shapes, it is necessary to establish that all corresponding angles and sides are
equal.
56 CHAPTER 3. GEOMETRY

In geometry, congruence refers to the property of two triangles being identical in shape and size. When
two triangles are congruent, all corresponding angles and sides of the triangles are equal. This means that
they can be superimposed on each other by translations, rotations, and reflections.
Key properties and concepts:
• Side-Side-Side (SSS) Congruence: If the three sides of one triangle are congruent to the three sides
of another triangle, then the triangles are congruent. This property is known as the Side-Side-Side
Congruence Theorem.
• Side-Angle-Side (SAS) Congruence: If one pair of corresponding sides of two triangles is congru-
ent, and the included angles are congruent, then the triangles are congruent. This property is known
as the Side-Angle-Side Congruence Theorem.

• Angle-Side-Angle (ASA) Congruence: If one pair of corresponding angles of two triangles is


congruent, and the included side is congruent, then the triangles are congruent. This property is
known as the Angle-Side-Angle Congruence Theorem.
• Angle-Angle-Side (AAS) Congruence: If two pairs of corresponding angles of two triangles are
congruent, and a pair of corresponding sides that includes one of the angles is congruent, then the
triangles are congruent. This property is known as the Angle-Angle-Side Congruence Theorem.
• Right Angle Hypotenuse Side (RHS) Congruence: Two right triangles are congruent, if the
hypotenuse and one side of one triangle are respectively equal to the hypotenuse and one side of the
other triangle.
3.2. QUADRILATERALS 57

3.2 QUADRILATERALS
3.2.1 Convex Quadrilaterals
A convex quadrilateral is a four-sided polygon (quadrilateral) where all of its interior angles are less than
180 degrees, and all its vertices point outward. In other words, all the corners of a convex quadrilateral
”stick out” and do not fold inward. Properties of Convex Quadrilaterals: Interior Angles: The sum of the
interior angles of a convex quadrilateral is always 360 degrees. Diagonals: A convex quadrilateral has two
diagonals, i.e., line segments joining non-adjacent vertices. In a convex quadrilateral, both diagonals lie
inside the polygon. Opposite Angles: The opposite angles of a convex quadrilateral are equal in measure.
For example, if we label the angles as A, B, C, and D, then angle A is equal to angle C, and angle B is
equal to angle D. Consecutive Angles: The consecutive angles of a convex quadrilateral are supplementary,
meaning the sum of two consecutive angles is 180 degrees. For example, angle A + angle B = 180 degrees,
angle B + angle C = 180 degrees, and so on. Parallel Sides: In a convex quadrilateral, opposite sides are
parallel. This property is true for parallelograms, which are a special type of convex quadrilateral. Equal
Opposite Sides: The opposite sides of a convex quadrilateral are equal in length. If we label the sides as AB,
BC, CD, and DA, then side AB is equal to side CD, and side BC is equal to side DA. Symmetry: A convex
quadrilateral can have one or more axes of symmetry, depending on its shape. For example, a square has
four axes of symmetry, while a rectangle has two. Area: The area of a convex quadrilateral can be calculated
using various formulas, depending on the information available about its sides and angles. One common
method is by dividing the quadrilateral into triangles and using the formula for triangle area.

Example problem and solution

Points E and F are on side BC of a convex quadrilateral ABCD with BE ¡ BF.Given that ∠BAE = ∠CDF
and ∠EAF = ∠F DE, prove that ∠F AC = ∠EDB.

Proof. Note that


∠EAF = ∠F DE

implies that AEF D is cyclic.


It suffices to show that ABCD is cyclic.
Note that
∠ADC = ∠ADF + ∠F DC

, so we have
∠ABC + ∠ADC = ∠ABC + ∠ADF + ∠F DC

. Then,
∠ABC = ∠AEF − ∠BAE

so it follows that
∠ABC + ∠ADC = ∠ABC + ∠ADF + ∠F DC

= ∠AEF − ∠BAE + ∠ADF + ∠F DC

= ∠AEF + ∠ADF

= 180

, which shows that ABCD is cyclic, as desired.


58 CHAPTER 3. GEOMETRY

3.2.2 Cyclic quadrilaterals


A cyclic quadrilateral is a four-sided polygon (quadrilateral) whose four vertices all lie on a single circle.
In other words, if you draw a circle that passes through all four vertices of the quadrilateral, it is said to
be a cyclic quadrilateral. Properties of Cyclic Quadrilaterals: Circumcircle: All four vertices of a cyclic
quadrilateral lie on the circumference of a single circle, known as the circumcircle of the quadrilateral.
Opposite Angles: The opposite angles of a cyclic quadrilateral are supplementary, meaning the sum of two
opposite angles is 180 degrees. If we label the angles as A, B, C, and D, then angle A + angle C = 180
degrees, and angle B + angle D = 180 degrees. Cyclic Quadrilateral Theorem: The sum of the measures of
any pair of opposite angles in a cyclic quadrilateral is 180 degrees. This theorem is sometimes referred to
as the ”opposite angles of a cyclic quadrilateral add up to 180 degrees.” Diagonals and Circumcircle: The
intersection point of the diagonals of a cyclic quadrilateral lies on the circumcircle of the quadrilateral. In
other words, the two diagonals meet on the circle that passes through all four vertices. Angle between Side
and Tangent: The angle between one side of a cyclic quadrilateral and the tangent to the circumcircle at the
opposite vertex is equal to the opposite angle inside the quadrilateral. For example, if we draw the tangent to
the circumcircle at vertex A, then angle BCD = angle A. Ptolemy’s Theorem: Ptolemy’s theorem relates the
side lengths and diagonals of a cyclic quadrilateral. For a cyclic quadrilateral with side lengths AB, BC, CD,
and DA, and diagonals AC and BD, Ptolemy’s theorem states: AB · CD + BC · DA = AC · BD. Concyclic
Points: The vertices of a cyclic quadrilateral are called concyclic points because they all lie on the same circle.
Area: The area of a cyclic quadrilateral canp be calculated using various methods, including Brahmagupta’s
formula for cyclic quadrilaterals: Area = [(s − a)(s − b)(s − c)(s − d), where s is the semiperimeter, and
a, b, c, and d are the side lengths.

3.2.3 Solved example


Let ABC be a triangle and D be the foot of the altitude from A. Let E and F be on a line passing through
D such that AE is perpendicular to BC, AF is perpendicular to CF, and E and F are different from D. Let
M and N be the midpoints of the line segments BC and EF, respectively. Prove that AN is perpendicular
to NM.
solution
Proof. Note that ABED and AF CD are cyclic quadrilaterals. It follows that ABC ∼ AEF since

∠ABD = ∠AED

and
∠AF D = ∠ACD
. Similarly, we can show that
ABM ∼ AEN
since
AB BC 2BN BN
= = =
AE EF 2EM EM
3.3. CIRCLE 59

Therefore,
∠AN D = ∠AM D
and it follows that ANMD is cyclic. Therefore

∠AN M = 180 − ∠ADM = 90

as desired.

3.3 CIRCLE
3.3.1 Tangents and its properties
In geometry, a tangent is a line that touches a curve or a circle at a single point, without intersecting it.
The point where the tangent touches the curve or circle is called the point of tangency. Tangents have
• Tangent Line to a Circle: If a line is tangent to a circle at a particular point, it is perpendicular to the
radius of the circle that passes through that point. This property is essential for constructing tangents
to circles.
• Tangent and Radius: The tangent to a circle at a given point is perpendicular to the radius drawn to
that point. This property follows from the fact that the tangent and radius form a right angle at the
point of tangency.
• Unique Tangent: A circle can have only one tangent at any given point on its circumference. This
uniqueness property ensures that there is only one line that touches the circle at a specific point without
intersecting it.
• Tangent Chord Angle Theorem: The angle between a tangent and a chord drawn from the point of
tangency is equal to the angle formed by the chord in the alternate segment of the circle.
• Common Tangents: Two circles can have four types of common tangents: external common tangents,
internal common tangents, direct common tangents, and transverse common tangents. External and
internal common tangents lie outside and inside both circles, respectively, while direct and transverse
common tangents touch one circle internally and the other externally.
• Length of Tangent Segments: If two tangents are drawn to a circle from a point outside the circle, the
lengths of the tangent segments are equal.
• Secant-Tangent and Chord-Tangent Theorems: If a secant and a tangent intersect at a point on a
circle, the product of the lengths of the whole secant and its external segment is equal to the square of
the length of the tangent segment. Similarly, if a chord and a tangent intersect at a point on a circle,
the product of the lengths of the whole chord and its external segment is equal to the square of the
length of the tangent segment.

3.3.2 Chords and its properties


Length of a Chord: The length of a chord can be calculated using the distance formula or by using the
properties of right triangles formed by the radius and the chord.
• Diameter as a Special Chord: A diameter is a special type of chord that passes through the center of
the circle. It is the longest chord in a circle, and its length is twice the length of a radius.
• Bisecting Chord: The perpendicular bisector of a chord passes through the center of the circle. It
divides the chord into two equal parts.
• Intersecting Chords Theorem: In a circle, if two chords intersect, the product of the segments of one
chord is equal to the product of the segments of the other chord. This theorem is often used to solve
problems involving intersecting chords.
60 CHAPTER 3. GEOMETRY

• Perpendicular Chords and Diameter: If two chords in a circle are perpendicular, one of them must be
a diameter. This property helps identify diameters when given perpendicular chords.
• Angle with the Center: The angle subtended by a chord at the center of the circle is twice the angle
subtended by the same chord at any point on the circumference (inscribed angle).

• Circle Chord Properties: If two chords in a circle are congruent, they are equidistant from the center
of the circle. Conversely, if two chords are equidistant from the center, they are congruent.
• Sagitta: The perpendicular distance from the center of the circle to a chord is called the sagitta. In a
semicircle, the sagitta is the radius of the circle.
• Circle Theorems with Tangents: There are several properties that relate tangents and chords in a
circle. For example, if a tangent and a chord intersect at a point on the circle, the angle between them
is equal to the angle formed by the chord in the alternate segment of the circle.

3.3.3 Circle Theorems


1. A chord is a straight line joining two points on the circumference of a circle. So AB is a chord.
3.3. CIRCLE 61

2. A tangent is a straight line that touches the circumference of a circle at only one point. The angle
between a tangent and the radius is 90°.

3. Two tangents on a circle that meet at a point outside the circle are equal in length. So AC = BC.
62 CHAPTER 3. GEOMETRY

4. The angle in a semicircle is a right angle. So angle ABC = 90°.

5. When two angles are subtended by the same arc, the angle at the centre of a circle is twice the angle at
the circumference. So ∠AOB = 2∠ACB.Anglessubtendedbythesamearcatthecircumf erenceareequal.T hismeansthata
∠ADBand∠CAD = ∠CBD.
3.4. INTRODUCTION TO CARTESIAN PLANE 63

7.
6. A cyclic quadrilateral is a quadrilateral with all four vertices on the circumference of a circle. Opposite
angles in a cyclic quadrilateral total 180°. So x + y = 180° and p + q = 180°.

8. The angle between a tangent and chord is equal to the angle in the alternate segment, this is known as the
alternate segment theorem. So ∠BAT = ∠ACB.

3.4 INTRODUCTION TO CARTESIAN PLANE


3.4.1 Introduction
The Cartesian plane, also known as the coordinate plane or Cartesian coordinate system, is a fundamental
concept in mathematics. It provides a way to represent and locate points in a two-dimensional space.

3.4.2 Key Features:


• Axes: The Cartesian plane consists of two perpendicular lines called the x-axis and y-axis. The x-axis
is horizontal, and the y-axis is vertical. They intersect at a point called the origin, typically labeled as
(0, 0).
• Quadrants: The plane is divided into four quadrants by the x-axis and y-axis. The quadrants are
numbered counterclockwise starting from the top right as I, II, III, and IV. Each quadrant has a
unique combination of positive and negative x and y values.
• Coordinates: Points in the Cartesian plane are represented by ordered pairs of numbers, written as (x,
y), where x is the horizontal distance from the y-axis (positive to the right, negative to the left), and
y is the vertical distance from the x-axis (positive upward, negative downward).
• Axes Units: The distance between consecutive units on the x-axis and y-axis is often equal, representing
the scale of measurement on the plane. However, the units on each axis can have different intervals.

3.4.3 How to find a point coordinates on cartesian plane


To find a point on the Cartesian plane, you need to know its coordinates, which consist of an x-coordinate
and a y-coordinate. Here’s how you can find a point on the Cartesian plane:
• Locate the x-coordinate: Determine the horizontal position of the point on the x-axis. If the x-
coordinate is positive, move to the right from the origin. If it is negative, move to the left from the
origin. The distance you move corresponds to the absolute value of the x-coordinate.
• Locate the y-coordinate: Determine the vertical position of the point on the y-axis. If the y-coordinate
is positive, move upward from the origin. If it is negative, move downward from the origin. The
distance you move corresponds to the absolute value of the y-coordinate.
64 CHAPTER 3. GEOMETRY

• Intersection of x and y: The point on the Cartesian plane is located at the intersection of the horizon-
tal line representing the x-coordinate and the vertical line representing the y-coordinate. Mark this
intersection point.
• Label the point: Write the coordinates of the point as an ordered pair, in the form (x, y), where x
represents the x-coordinate and y represents the y-coordinate.
3.5. APPROACHES AND BASICS IN SOLVING OLYMPIAD PROBLEMS USING GEOMETRY 65

3.5 APPROACHES AND BASICS IN SOLVING OLYMPIAD


PROBLEMS USING GEOMETRY
3.5.1 Angle chasing
Angle chasing is a problem-solving technique in geometry that involves systematically using angle relation-
ships and properties to find unknown angles in a given geometric figure. This technique is particularly
useful when dealing with polygons, circles, and other geometric shapes where angles play a significant role.
Angle chasing often requires identifying angles formed by intersecting lines, parallel lines, angles in triangles,
angles in quadrilaterals, and angles in circles. The steps involved in angle chasing may vary depending on
the specific problem, but the general approach includes the following strategies:

• Identify Known Angles: Start by identifying any angles that are already given or can be easily deter-
mined from the problem statement. This may include angle measurements provided in the figure or
angles formed by intersecting lines or parallel lines.

• Use Angle Sum Properties: In polygons, such as triangles and quadrilaterals, the sum of the interior
angles is constant. For example, in a triangle, the sum of the three interior angles is always 180 degrees.
Utilize these angle sum properties to find unknown angles when some angles are given.

• Use Angle Relationships: Identify angle relationships within the figure, such as vertical angles, corre-
sponding angles, alternate interior angles, and supplementary angles. These relationships can help you
find missing angles by setting up equations and solving for the unknown values.

• Use Angle Properties in Circles: For problems involving circles, use the properties of central angles,
inscribed angles, and angles formed by intersecting chords and tangents. These properties provide
relationships between angles and arc measures, which can be used to find unknown angles.

• Extend Lines and Create Parallel Lines: In some cases, you may need to extend lines or create parallel
lines to form specific angle relationships that can aid in finding unknown angles.

• Work Methodically: Angle chasing can involve multiple steps and several angle relationships, so it’s
essential to work methodically and keep track of the angles you find along the way. Draw additional
lines or label angles as needed to help you visualize the relationships. Be Patient and Persistent: Angle
chasing can sometimes be challenging, and finding all the necessary angle relationships may take time
and multiple attempts. Be patient and persistent in your approach, and try different strategies if you
get stuck.

Chasing con’t
One of the most crucial skills that is necessary to master in order to be a good geometer is the skill of
angle chasing. In this meeting, I will go over the skills that are generally lumped under the “angle chasing”
category before giving an example as to what angle chasing is. The majority of this meeting will consist of
problem-solving, as many of the skills you probably already know.

What exactly is angle chasing?


Angle chasing usually consists of any of the following techniques:

• vertical angles

• sum of angles in a triangle

• congruent triangles

• similar triangles

• supplementary angles
66 CHAPTER 3. GEOMETRY

• complementary angles

• parallel lines and angles

• perpendicular lines

• angle bisectors

• angles intercepting arcs

However, the above techniques are not necessarily all-inclusive when it comes to angle chasing. An example
of this can be shown in the problem below.
Example 1 (MA0 1987). In the figure, O is the center of the circle, ∠EAD = 40◦ , and ∠EDd = 40◦ .
Find ∠DAB.

Solution. Note that from secant-secant stuff we get that ∠EAD = 21 (CF d − EDd). Since ∠EAD and
∠EDd are both 40◦ , we can substitute to get 40◦ = 21 (CF d − 40◦ ) ⇒ CF d = 120◦ . Next, note that since
CDd is a diameter, we get that DF d = 180◦ − CF d = 60◦ , so ∠DCB = 30◦ . Finally, since BE ⊥ AC
and AF ⊥ BC, D is the orthocenter of △ABC. This means that CD ⊥ AB, so ∠DAB = 90◦ − ∠B =
∠DCB = 30◦ . (Don’t worry, most of the problems in the problem section do not require knowledge of the
orthocenter!)

Examples on topic
1. △ ABC is an isosceles triangle with AB=BC and ∠ABD = 600 , ∠DBC = 200 and

∠DCB = 100

. Find ∠BDA.
Solution Let E be the circumcenter of BCD . Then

∠BED = 2∠BCD = 200

and
∠DEC = 2∠DBC = 400
. Hence
∠BEC = 600
. This and BE=EC
shows that BEC is equilateral. So BC=BE and

∠CBE = 600
3.5. APPROACHES AND BASICS IN SOLVING OLYMPIAD PROBLEMS USING GEOMETRY 67

. By assumption AB=BC , so AB=BE and


1 1
∠BEA = 900 − ∠ABE = 900 − · 140[ 0] = 200 = ∠BED
2 2
. Therefore A,D,E are collinear and we find

∠BDA = 1800 − ∠EDB = ∠BED + ∠DBE = 200 + 800 = 1000

.
68 CHAPTER 3. GEOMETRY

Exercises
1. In △ABC, AB = AC and ∠A = 40◦ . The bisector from ∠B intersects AC at point D. What is
∠BDC?
2. In the adjoining figure, ABCD is a square, ABE is an equilateral triangle, and point E is outside
square ABCD. What is the measure of ∠AED?

3. (MATHCOUNTS 1986) △ABC is an isosceles triangle such that AC = BC. △CBD is an isosceles
triangle such that CB = DB. BD meets AC at a right angle. If ∠A = 57◦ , what is ∠D?
4. A square is located in the interior of a regular hexagon, and certain vertices are labeled as shown.
What is the degree measure of ∠ABC?

5. The keystone arch is an ancient architectural feature. It is composed of congruent isosceles trapezoids
fitted together along the non-parallel sides, as shown. The bottom sides of the two end trapezoids are
horizontal. In an arch made with 9 trapezoids, let x be the angle measure in degrees of the larger
interior angle of the trapezoid. What is x?
3.5. APPROACHES AND BASICS IN SOLVING OLYMPIAD PROBLEMS USING GEOMETRY 69

6. [M ath League HS 2013 − 2014/2009 − 2010/1994 − 1995] In a certain quadrilateral, the three shortest
sides are congruent, and both diagonals are as long as the longest side. What is the degree measure of
the largest angle of this quadrilateral?
7. Let △ABC be a right triangle with a right angle at C. Let D and E be the feet of the angle bisectors
from A and B to BC and CA, respectively. Suppose that AD and BE intersect at point F . Find
∠AF B.
8. (AHSME 1990) An acute isosceles triangle △ABC is inscribed in a circle. Through B and C, tangents
to the circle are drawn, meeting at point D. If ∠ABC = ∠ACB = 2∠D, find the measure of ∠A.
9. (BMO1 1995) Triangle ABC has a right angle at C. The internal bisectors of angles BAC and ABC
meet BC and CA at P and Q respectively. The points M and N are the feet of the perpendiculars from
P and Q to AB. Find ∠M CN .
10. (ELMO SL 2013, Owen Goff) Let ABC be a triangle with incenter I. Let U, V and W be the intersections
of the angle bisectors of angles A, B, and C with the incircle, so that V lies between B and I, and
similarly with U and W. Let X, Y , and Z be the points of tangency of the incircle of triangle ABC
with BC, AC, and AB, respectively. Let triangle UVW be the David Yang triangle of ABC and let
XYZ be the Scott Wu triangle of ABC. Prove that the David Yang and Scott Wu triangles of a triangle
are congruent if and only if ABC is equilateral.
11. (IMO 1990) Chords AB and CD of a circle intersect at a point E inside the circle. Let M be an interior
point of the segment EB. The tangent line at E to the circle through D, E, and M intersects the lines
−−→ −→
BC and AC at F and G, respectively. If AM EG
AB = t, find EF in terms of t. (Note: You may want to
explore the next problem a bit before attempting this one.)
12. An exploration of cyclic quadrilaterals.
(a) Let ABCD be a quadrilateral inscribed inside a circle. Prove that:
i. ∠ABC + ∠ADC = 180◦ .
ii. ∠ABD = ∠ACD.
(b) Assume that the converse is true, i.e., if any one of the two properties above holds, then the
quadrilateral is cyclic. This is a very useful tool, as it allows you to switch back and forth between the
two angle properties/equalities. Using this knowledge, prove the following statements:
i. Let ABC be a right triangle with the right angle at B. Let D be any point on AB, and let E be the
foot of the perpendicular from D to AC. Prove that ∠DBE = ∠DCE.
ii. [Canada 1986] A chord ST of constant length slides around a semicircle with diameter AB. M is the
midpoint of ST and P is the foot of the perpendicular from S to AB. Prove that angle SPM is constant
for all positions of ST.
iii. [Sharygin 2009] Given triangle ABC. Points M, N are the projections of B and C to the bisectors of
angles C and B respectively. Prove that line MN intersects sides AC and AB in their points of contact
with the incircle of ABC.

3.5.2 Working backwards


A common stratagem, when trying to prove that a given point has a desired property, is to construct a
phantom point with the desired property, then reason backwards to show that it coincides with the original
point. We illustrate this point with an example.

Theorems
Suppose the triangles △ABC and △AB1 C1 are directly similar. Then the points A, B, C, BB1 ∩ CC1 lie
on a circle.
Since we want to show that BB1 ∩ CC1 lie on the circle through A, B, C, and analogously on the circle
through A, B1 , C1 , we define the point P to be the intersection of these two circles. Then

∠AP B = ∠ACB, ∠AC1 B1 = ∠AP B1 .


70 CHAPTER 3. GEOMETRY

So, P lies on the line ∠BB1 , and similarly on the line ∠CC1 .

EXERCISES ON TOPIC
1. (IMO 1994/2) Let △ABC be an isosceles triangle with AB = AC. Suppose that
←−→ ←→ ←→
(i) M is the midpoint of BC and O is the point on the line AM such that OB is perpendicular to AB;
(ii) Q is an arbitrary point on the segment BC different from B and C;
←→ ←→
(iii) E lies on the line AB and F lies on the line AC such that E, Q, F are distinct and collinear.
←→ ←→
Prove that OQ is perpendicular to EF if and only if QE = QF .
2. (USAMO 2005/3) Let △ABC be an acute-angled triangle, and let P and Q be two points
on side BC. Construct point C1 in such a way that convex quadrilateral AP BC1 is cyclic,
←−→ ←→ ←→
QC1 ∥ CA, and C1 and Q lie on opposite sides of line AB. Construct point B1 in such a way
←−→ ←→
that convex quadrilateral AP CB1 is cyclic, QB1 ∥ BA, and B1 and Q lie on opposite sides of line
←→
AC. Prove that points B1 , C1 , P , and Q lie on a circle.
3. (Morley’s theorem) Let △ABC be a triangle, and for each side, draw the intersection of the
two angle trisectors closer to that side. (That is, draw the intersection of the trisectors of A
and B closer to AB, and so on.) Prove that these three intersections determine an equilateral
triangle.

3.6 PROBLEMS
Facts you should know
1. Let ABC be a triangle and extend BC past C to D. Show that ∠ACD = ∠BAC + ∠ABC.
2. Let ABC be a triangle with ∠C = 90. Show that the circumcenter is the midpoint of AB.
3. Let ABC be a triangle with orthocenter H and feet of the altitudes D, E, F . Prove that H is the
incenter of △DEF .
4. Let ABC be a triangle with orthocenter H and feet of the altitudes D, E, F . Prove that (i) A, E, F, H
lie on a circle with diameter AH, and (ii) B, E, F, C lie on a circle with diameter BC.
5. Let ABC be a triangle with circumcenter O and orthocenter H. Show that ∠BAH = ∠CAO.
6. Let ABC be a triangle with circumcenter O and orthocenter H, and let AH and AO meet the circum-
circle at D and E, respectively. Show (i) that H and D are symmetric with respect to BC, and (ii)
that H and E are symmetric with respect to the midpoint of BC.
7. Let ABC be a triangle with altitudes AD, BE, and CF . Let M be the midpoint of side BC. Show
that M E and M F are tangent to the circumcircle of △AEF .
8. Let ABC be a triangle with incenter I, A-excenter Ia , and D the midpoint of arc BC not containing
A on the circumcircle. Show that DI = DIa = DB = DC.
9. Let ABC be a triangle with incenter I and D the midpoint of arc BC not containing A on the
circumcircle. Define E and F similarly. Show (i) that I is the orthocenter of △DEF , and (ii) that
A, B, C are the reflections of I across EF, F D, DE respectively.
10. Let ABC be a triangle with incenter I and excenters Ia , Ib , Ic . Prove that in triangle Ia Ib Ic , A, B, C
are the feet of the altitudes and I is the orthocenter.
11. Let ABC be a triangle with incenter I, and whose incircle is tangent to sides BC, AC, AB at D, E, F
respectively. Let M, N be midpoints of BC, AC respectively. Prove that EF, BI, M N concur.
3.6. PROBLEMS 71

12. (Simson Line) Let ABC be a triangle and D a point on its circumcircle. Prove that the feet of the
perpendiculars from D to lines AB, AC, and BC are collinear.
13. (Nine Point Circle) Let ABC be a triangle with orthocenter H, altitudes AA1 , BB1 , and CC1 , and mid-
points A2 , B2 , C2 . Let the midpoints of AH, BH, CH be A3 , B3 , C3 . Show that A1 , A2 , A3 , B1 , B2 , B3 , C1 , C2 , C3
lie on a circle.

Problems
1. In parallelogram ABCD, let the bisector of ∠BCD intersect lines AB and AD at E and F , respectively.
Prove that BE = AD and DF = AB.
2. Let ABC be a right triangle, let ∠C be the right angle, and let D be the foot of the altitude from C.
Prove that the circumcenters of ACD, CBD, and ABC form a triangle that is similar to ABC.
3. ω1 , ω2 are two circles intersecting at P and Q. Let A be a variable point on ω1 , and B, C be the
intersections of AP, AQ with ω2 . Show that the size of BC is independent of A.
4. (British Math Olympiad 2000) Two intersecting circles C1, C2 have a common tangent P Q with P on
C1 and Q on C2. The two circles intersect at M, N , where P Q is nearer to M . The line P N meets
the circle C2 again at R. Prove that M Q bisects ∠P M R.
5. (”Largely Artistic Math Olympiad”) Two circles ω1, ω2 intersect at P, Q. If a line intersects ω1 at
A, B and ω2 at C, D such that A, B, C, D lie on the line in that order, and P and Q lie on the same
side of the line, compute ∠AP C + ∠BQD.
6. Let P be a point inside circle ω. Consider the set of chords of ω that contain P . Prove that their
midpoints all lie on a circle.
7. (British Math Olympiad 2005) The triangle ABC, where AB < AC, has circumcircle S. The perpen-
dicular from A to BC meets S again at P . The point X lies on the line segment AC, and BX meets
S again at Q. Show that BX = CX if and only if P Q is a diameter of S.
8. (Own) In triangle ABC, M and N are midpoints of AC and AB respectively. Point D is on BC such
that M D = M C. Extend lines M D and N D to meet AB and AC at F and E, respectively. Prove
that EF is perpendicular to BC.
9. (ELMO 2012) In acute triangle ABC, let D, E, F denote the feet of the altitudes from A, B, C, respec-
tively, and let ω be the circumcircle of △AEF . Let ω1 and ω2 be the circles through D tangent to ω
at E and F respectively. Show that ω1 and ω2 meet at a point P on BC other than D.
10. (USA(J)MO 2010) Let AXY ZB be a convex pentagon inscribed in a semicircle of diameter AB.
Denote by P, Q, R, S the feet of the perpendiculars from Y onto lines AX, BX, AZ, BZ respectively.
Prove that the acute angle formed by lines P Q and RS is half the size of ∠XOZ, where O is the
midpoint of segment AB.
11. (IMO Shortlist 2010) Let ABC be an acute triangle with D, E, F the feet of the altitudes lying on
BC, CA, AB respectively. One of the intersection points of the line EF and the circumcircle is P . The
lines BP and DF meet at point Q. Prove that AP = AQ.
12. Let ABCDE be a convex pentagon such that BCDE is a square with center O and ∠A = 90. Prove
that AO bisects ∠BAE.
13. (IMO 2006) Let ABC be a triangle with incenter I. A point P in the interior of the triangle satisfies
∠P BA + ∠P CA = ∠P BC + ∠P CB. Show that AP ≥ AI and that equality holds if and only if P = I.
14. (IMO 2002) The circle S has center O, and BC is a diameter of S. Let A be a point of S such that
∠AOB < 120◦ . Let D be the midpoint of the arc AB which does not contain C. The line through O
parallel to DA meets the line AC at I. The perpendicular bisector of OA meets S at E and F . Prove
that I is the incenter of the triangle CEF .
72 CHAPTER 3. GEOMETRY

15. (Balkan MO 2012) Let A, B, and C be points lying on a circle Γ with center O. Assume that ∠ABC >
90◦ . Let D be the point of intersection of the line AB with the line perpendicular to AC at C. Let l
be the line through D which is perpendicular to AO. Let E be the point of intersection of l with the
line AC, and let F be the point of intersection of Γ with l that lies between D and E. Prove that the
circumcircles of triangles BF E and CF D are tangent at F .

16. (APMO 2007) Let ABC be an acute-angled triangle with ∠BAC = 60◦ and AB > AC. Let I be the
incenter, and H the orthocenter of the triangle ABC. Prove that 2∠AHI = 3∠ABC.
17. In scalene triangle ABC, H, I, and O are the orthocenter, incenter, and circumcenter respectively.
Prove that one of the angles of the triangle is 60◦ if and only if IH = IO.

18. (EGMO 2012) Let ABC be an acute-angled triangle with circumcircle Γ and orthocenter H. Let K
be a point of Γ on the other side of BC from A. Let L be the reflection of K in the line AB, and
let M be the reflection of K in the line BC. Let E be the second point of intersection of Γ with the
circumcircle of triangle BLM . Show that the lines KH, EM, and BC are concurrent.
19. (CGMO 2012) In triangle ABC, AB = AC. Point D is the midpoint of side BC. Point E lies outside
the triangle ABC such that CE ⊥ AB and BE = BD. Let M be the midpoint of segment BE. Point
F lies on the minor arc AD of the circumcircle of triangle ABD such that M F ⊥ BE. Prove that
ED ⊥ F D.
20. In triangle ABC, let D ∈ BC, E ∈ AC, F ∈ AB be the points of tangency of the incircle to the
sides. Let I be the incenter. The parallel line through A to BC intersects DE and DF at M and N
respectively. Let L and T be the midpoints of the segments N D and DM . Show that A, L, I, T lie on
a circle.
21. (EGMO 2012) Let ABC be a triangle with circumcenter O. The points D, E, F lie in the interiors of
the sides BC, CA, AB respectively, such that DE ⊥ CO and DF ⊥ BO. Let K be the circumcenter
of triangle AF E. Prove that the lines DK and BC are perpendicular.

22. (IMO 2010) Given a triangle ABC, with I as its incenter and Γ as its circumcircle, AI intersects Γ
again at D. Let E be a point on arc BDC, and F a point on the segment BC, such that ∠BAF =
∠CAE < 12 ∠BAC. If G is the midpoint of IF , prove that the intersection of lines EI and DG lies on
Γ.
23. (Romanian TST 1996) Let ABCD be a cyclic quadrilateral and let M be the set of incenters and
excenters of the triangles BCD, CDA, DAB, ABC (16 points in total). Prove that there are two sets
K and L of four parallel lines each, such that every line in K ∪ L contains exactly four points of M .
Chapter 4

COMBINATORICS

4.1 COUNTING METHODS


4.1.1 Introduction
Combinatorics is an area of mathematics primarily concerned with counting, both as a means and an end
in obtaining results, and certain properties of finite structures. It is closely related to many other areas
of mathematics and has many applications ranging from logic to statistical physics and from evolutionary
biology to computer science.

As an area can be described by the types of problems it addresses, combinatorics is involved with:

• The enumeration (counting) of specified structures, sometimes referred to as arrangements or configu-


rations in a very general sense, associated with finite systems,

• The existence of such structures that satisfy certain given criteria

• The construction of these structures, perhaps in many ways, and

• Optimization: finding the ”best” structure or solution among several possibilities, be it the ”largest”,
”smallest” or satisfying some other optimality criterion.

In this book, our main interest is to teach you different methods of counting and providing you with an
intuition for approaching counting problems.

4.1.2 Fundamental principles of counting


Addition Principle (AP)
Let A1 and A2 be disjoint events, that is events having no common outcomes, with n1 and n2 possible
outcomes for each event , respectively. Then the total number of outcomes for the event
”A1 or A2 ” is n1 +n2 . Note that the events must be disjoint, that is they must not have common outcomes
for this principle to be applicable.

Example Suppose there is this list of drinks on the table: Fanta, Juice and Water. How many selec-
tions does Kamanzi have for picking one drink? Suppose there is this list of drinks on the table: Fanta, Juice
and Water. How many selections does Kamanzi have for picking one drink?

Solution: In this case, an event is ”selecting a drink”. There is 1 outcome for the Fanta event, 1 out-
come for the juice event and 1 outcome for the water event. According to the addition principle there are
:1+1+1 = 3 possible selections. This addition principle can be generalized for more than two events.

73
74 CHAPTER 4. COMBINATORICS

General Addition Principle:


LetA1 , A2 , ... Ak be disjoint events with n1 ,n2 , ... nk possible outcomes, respectively. Then the total
number of outcomes for the event ”A1 or A2 or ... or Ak ” is n1 +n2 + ... +nk .

Product principle
Let A1 and A2 be events with n1 and n2 possible outcomes, respectively. Then the total number of
outcomes for the sequence of the two events is n1 × n2 .

General Multiplication Principle:

Let A1 , A2 , ... Ak be events with n1 ,n2 , ... nk possible outcomes, respectively. Then the total number
of outcomes for the sequence of these k events is n1 × n2 × ... × n2 .
Example: Suppose that there are three major auto routes from Washington DC to Chicago, and five from
Chicago to Los Angeles. Then there are 3 × 5= 15 major routes from Washington DC to Los Angeles.
INCLUSION-EXCLUSION PRINCIPLE
Principle of Inclusion and Exclusion is an approach which derives the method of finding the number of
elements in the union of two finite sets. This is used to solve combinations and probability problems when
it is necessary to find a counting method, which makes sure that an object is not counted twice.
A frequently occurring problem is to determine the size of the union or intersection of a number of sets, as
in the example below:

At a certain university, all second-year science students may choose either mathematics, or physics, or
both. The mathematics course is attended by 50 students, the physics course by 30 students. 15 students
attend both courses. How many second-year science students are there?

SOLVED EXAMPLE: There are 350 farmers in a large region. 260 farm beetroot, 100 farm yams,
70 farm radish, 40 farm beetroot and radish, 40 farm yams and radish, and 30 farm beetroot and yams.
Let B, Y, and R denote the set of farms that farm beetroot, yams and radish respectively. Determine the
number of farmers that farm both beetroot, yams, and radish.
Solution:

The letters for denoting the sets have already been provided in the question itself (unlike the above ex-
ample). We may therefore note the cardinality straight away:
|U | = 350; |B| = 260; |Y | = 100; |R| = 70; |B ∩ R| = 40; |Y ∩ R| = 40 and |B ∩ Y | = 30 We need to determine
the cardinality of the intersection of all three sets, which is |B ∩ Y ∩ R|. This is the unknown which we can
assign determine algebraically.. Populate a Venn diagram with the given information. Use x to represent
|B ∩ Y ∩ R|.
Let x farmers farm beetroot, yams, and radish. That is, let |B ∩ Y ∩ R| = x
Now solve for x algebraically:
|U | = 350 = 190 + x + (30–x) + x + (40–x) + (40–x) + 30 + x + x–10
350 = 320 + x
x = 30
Therefore, 30 farmers farm beetroot, yams, and radish.
COUNTING NUMBERS IN A LIST
How many numbers are in the list 2013, 2014, ....., 2497? Rule: In a list of consecutive numbers, a,
a+1,...,b there are b-a+1 = b-(a-1) numbers.
4.1. COUNTING METHODS 75

4.1.3 Warm-up
1. There are 9 boys and 8 girls in a class. In how many ways can the teacher
(a) pick a team of 3 students?
(b) pick a team of 6 students?
(c) pick a team of 6 students that is of one gender?
d) pick a team of 3 boys and 3 girls?
(e) choose a team captain and a vice-captain from the students?
(f) choose a team captain (of any gender) and a vice-captain ( must be a female) from the students?
(g) line the students up in a line?

2. Alphabet things
(a) How many ways can you rearrange the letters “AABC”?
(b) How many ways can you rearrange the letters “AAABBCCC”?
9!
( c) Can you make a question so that the answer is 2!2!3!?
(d) You have the letters “AAABBBCCC”. How many 3 letter words can you make using these letters?
(e) You have the letters “AAABBBCCC”. How many 4 letter words can you make using these letters?

3. A child has 5 red marbles, 4 white marbles, and 5 green marbles. He wants to bring some of his marbles
to school.
(a) He places 3 marbles in a box, order matters, how many ways can he do this? (Bonus question for
later, what if order does not matter?)
(b) How many different combinations of marbles can he bring to school? (Any number)
(c) How many combinations if he wants to bring a different number of each color?
(d) He takes all his marbles and lines them up on the floor, how many patterns can he make?

4. x1 ,x2 ,x3 ,... are all non-negative integers (i.e. from the set 0,1,2,...). How many solutions to the follow-
ing?
(a) x1 +x2 = 11
(b) x1 +x2 = 2
(c) x1 +x2 = n
(d)x1 +x2 +x3 =83
(e) x1 and x2 are positive integers, how many solutions to x1 +x2 = 10
(f) x1 , x2 and x3 are positive integers how many solutions to x1 +x2 +x3 =10
(g) x1 , x2 , x3 and x4 are positive integers, how many solutions to x1 +x2 +x3 +x4 = 40
(h)x1 ,x2 and x3 are all integers larger than -3. How many solutions to the equation x1 +x2 +x3 = 20?

5. There are 18 boys on a squad. The coach must pick a team of 11 to play on the field. 3 students-
Jules, Isaac and Pacifique all refuse to play with each other. (i.e. if Jules is on the field, then Isaac
andPacifique aren’t). In how many ways can the coach pick a team?

6. How many binary numbers exist that are made up of 5 digits? (for example 00111 and 10101)

7. I have 8 students, some will get pass and some will get fail. How many different results are possible?
(First try: each student gets either a pass or a fail. Then try to count based on number of passes - i.e.
case where there is 0 passes, plus when there is 1 pass plus... )

8. a)I have 10 identical soccer balls to give to three students. In how many ways can I do this?
b)What if the soccer balls were distinct?

9. You have 100 marbles each of colors blue, green, red and yellow. You choose 10 marbles. In how many
ways can you do this? (order of color /choosing does not matter)
76 CHAPTER 4. COMBINATORICS

10. A magic number is a number made up of 10 digits: 0,1,2,3,4,5,6,7,8 and 9. The n first digits makes
a number which is divisible by n. For example 1264502783 has 1 which is divisible by 1, 12 which is
divisible by 2, 126 which is divisible by 3, 1264 which is divisible by 4 and 12645 which is divisible by
5. But the condition is not satisfied for all remaining numbers as simply this number number is not
divisible by 10. How many magic numbers can you make?

11. I have an 8 × 8 grid, how many paths from the bottom left corner to the top right corner? What
about for a 7 × 6 grid?
12. A cow starting from point (0,0) jumps either to point (x+1,y+2) or to (x+2, y+1). How many paths
are there from (0,0) to (15,15) using those steps given above in any order?
4.2. THE PRINCIPLE OF INDUCTION 77

4.2 THE PRINCIPLE OF INDUCTION


The principe of induction:Let a be an integer , and let p(n) be a statement ( or proposition) about
n for each n ≥ a. The principle of induction is a way of proving that p(n) is true for all integers n ≥ a.
It works in three steps:
(a) Base case: Prove that p(a) is true.
(b) Hypothesis step: Assume that p(k) is true for some integer k which is greater than a.
(c) nductive step: Use the fact that p(k) is true to prove that p(k+1) is also true. After confirming
that for any p(k) which is true leads to p(k+1) which is true, we can conclude that p(n) is true
for all integers n ≥ a.

This principle of induction is very useful in problem solving, especially when we observe a pattern and
want to prove it. The trick to using the principle of induction properly is to spot how to use p(k) to
prove p(k+1). Sometimes this must be done rather ingeniously.

n(n+1)
Example: Prove that: 1 + 2 + 3 + 4 + 5 + ... + n = 2 (The sum of n first natural numbers)

PROOF

Step 1: Base case : For n=1;


LHS=1
RHS= 1(1+1)
2 = 1 It is true for n=1 since LHS=RHS=1
Step 2: Hypothesis step : Assume that it is true for some positive integer k, so that p(k)=1+2+3+...+k= k(k+1)
2
then we have to prove that p(k+1) is also true.
Step 3: Inductive step
p(k+1)=1+2+3+...+k+(k+1)=p(k)+(k+1)= k(k+1) 2 + (k + 1) = (K + 1)( k2 + 1) = (k+1)(k+2)
2
Hence p(n) is true for all natural numbers n by the principle of induction.

Example 3: Find the sum of n first odd numbers.


Solution:
We are asked to find the series 1+3+5+7+9+...+(2n-1). Note that this time we are not told the formula
that we have to prove ; We have to find it ourselves ! Let’s try some small numbers and see if a pattern
emerges.
1=1; 1+3=4; 1+3+5=9; 1+3+5+7=16; 1+3+5+7+9=25,... We conjecture(guess) that the sum of the
first n odd numbers is equal to n2 .
Now , let’s prove our statement using induction principle.
p(n)=1+3+5+...+(2n-1)=n2

Step1: Base case: For n=1, p(1)=1=12


Step 2: Hypothesis step: Assume that for some positive integer k, p(k)=1+3+5+...+(2k-1)=k 2 , the
we have to prove that p(k+1) is also true.
Step 3: Inductive step: p(k+1)=1+3+5+...+(2k-1)+(2k+1)=k2+2k+1=(k + 1)2

Therefore,by the principle of induction, p(n)=1+3+5+...+(2n-1)=n2 for all odd n.


Example 3:
Show that 2n+2 + 32n+1 is divisible by 7 for all positive integers n.
SOLUTION
Step 1: Base case: For n=1, p(1)=21+2 + 32+1 = 35 Which is divisible by 7.
Step 2: Hypothesis case: Assume p(k) is true for some positive integer k such that7|2k+2 + 32k+1 then
, let’s prove if p(k+1) is also true.
Step 3: Inductive step:
78 CHAPTER 4. COMBINATORICS

p(k+1)=2(k+1)+2 + 32(k+1)+1
=2.2k+2 + 32 .32k+1
=2[2k+2 + 32k+1 ] + 7.32k+1
=2p(k) + 7.32k+1
From our assumption in the hypothesis step, we have assumed that 7|p(k) therefore, p(k)=7m, m being
an integer. It implies that 2p(k)=14m then,
p(k+1)=14m + 7.32k+1 = 7(2m + 32k+1 ) which is obviously divisible by 7, hence by induction,
We have proved that for all integer n ,7|2n+2 + 32n+1

Notice: a|b means a divides b or b is divisible by a.

4.2.1 Warm-up
n(n+1)(2n+1)
1. Show that :12 + 22 + 32 + 42 + ... + n2 = 6 ∀n ∈ N.
2. Prove that 4n − 1 is divisible by 3 ∀n ∈ Z+ .
3. 1+5+9+...+(4n – 3) = n(2n–1) for all natural numbers n.

4. Show that ∀n ∈ N :2+4+6+ ... +2n = n2 + n.


5. Verify if 2n < (n + 2)!∀n ∈ N.
1 1 1 1
6. Prove that n+1 + n+2 + n+3 + . . . + 2n > 13
24 ∀n ∈ N and n > 1.

7. Prove that n < √11 + √12 + √13 + ... + √1n for all natural number n ≥ 2.

n5 n3 7n
8. Show that ∀n ∈ N : 5 + 3 + 15 is a natural number .
1 1 1 1 n+1
9. Prove that (1 − 22 ) × (1 − 32 ) × (1 − 42 ) × ... × (1 − n2 ) = 2n for all natural numbers n ≥ 2.
10. Using induction principle, prove the statement below.
Let x be a positive real number. Then for any natural number n, we have (1 + x)n ≥ 1 + nx.
4.3. SEQUENCES AND SERIES 79

4.3 SEQUENCES AND SERIES


4.3.1 Sequences
Definitions
Sequences are ordered lists of numbers (called ”terms”), like 2,5,8. Some sequences follow a specific pat-
tern that can be used to extend them indefinitely. For example, 2,5,8 follows the pattern ”add 3,” and now
we can continue the sequence. Sequences can have formulas that tell us how to find any term in the sequence.

MAIN TYPES OF SEQUENCES


• Arithmetic Sequences.
• Geometric Sequence.
• Fibonacci Sequence

ARITHMETIC SEQUENCE/ ARITHMETIC PROGRESSION


Arithmetic Progression (AP) is a sequence of numbers in order, in which the difference between any two
consecutive numbers is a constant value. It is also called Arithmetic Sequence. For example, the series
of natural numbers: 1,2,3,4,5,6,. . . is an Arithmetic Progression, which has a common difference between
two successive terms (say 1 and 2) equal to 1 (2 -1). Even in the case of odd numbers and even numbers,
we can see the common difference between two successive terms will be equal to 2.

Finding of the nth term in the AP: Let an be the nth term of the Arithmetic Progression.
We have:
a1 = a
a2 = a + d
a3 = a + 2d
.
.
.
an = a + (n − 1)d
[ Try to prove it using induction though it is obvious]
4.3.2 Check understanding
1. What is the 20th term in the following sequence: 1,9,17,25,...
2. The first term an AP is 23 and the 16th term of that AP is 890. What is the common difference of
that AP.
3. Derive the formula for the sum of n consecutive terms of an AP whose first term is a and common
difference is d.

GEOMETRIC PROGRESSION

In Maths, Geometric Progression (GP) is a type of sequence where each succeeding term is produced by
multiplying each preceding term by a fixed number, which is called a common ratio. This progression
is also known as a geometric sequence of numbers that follow a pattern.For example, 2, 4, 8, 16, 32,
64, . . . is a GP, where the common ratio is 2.
Mathematically, a GP is the one whose terms are in this order: a, ar, ar2 , ar3 , ..., arn−1

CHECK UNDERSTANDING
80 CHAPTER 4. COMBINATORICS

(a) Give two examples of geometric progressions.


(b) Derive the formula for the sum of n terms of the geometric progression whose first term is a and
the common ratio is r.
(c) Find the area of the shaded part, knowing that the length of the larger square is 1 unit length.
(Not drawn to scale)

4.3.3 Fibonacci sequence

Read carefully this story: ‘’ Fibonacci rabbit never die and can give birth without being married”
Fibonacci had a magic rabbit species. He started with one such a rabbit. This rabbit gives birth to one
young-one for the first time two months later after it was born. After the 2 months, the rabbit gives
birth to one young-one each month until forever because this rabbit never die. The rabbit doesn’t need
to be a male or a female in order to give birth the young-one. The young-ones also behave like their
parent rabbit.

CHECK UNDERSTANDING
(a) Demonstrate mathematically the information narrated in the story using your own approach.
(b) Starting at the birth of the first rabbit of fibonacci, how many rabbits will fibonacci have after 6
months?

In mathematics, the Fibonacci sequence is a sequence in which each number is the sum of the two
preceding ones. Numbers that are part of the Fibonacci sequence are known as Fibonacci numbers,
commonly denoted Fn . The sequence commonly starts from 0 and 1, although some authors start the
sequence from 1 and 1 or sometimes (as did Fibonacci) from 1 and 2. Starting from 0 and 1, the first
few values in the sequence are:0, 1, 1, 2, 3, 5, 8, 13, 21, 34, 55, 89, 144.

Warm-up

(a) The first, the second and the fourth terms of an AP are three first terms of a certain GP. If the
sum of the first 5 terms of that GP is 31, what is the sum of the first 7 terms of that AP?
(b) Three lengths of sides of a triangle forms an AP and the area of that triangle is 6. What are the
lengths of each side of that triangle? What type is that triangle ?
(c) (IrMO 2021) A sequence whose first term is positive has the property that any given term is the
area of an equilateral triangle whose perimeter is the preceding term. If the first three terms form
an arithmetic progression, determine all possible values of the first term.

4.4 PIGEON HOLE PRINCIPLE


4.4.1 Definition
If n + 1 objects are placed into n boxes, then some box contains at least 2 objects.
Proof: Suppose that each box contains at most one object. Then there must be at most n objects in
all. But this is false, since there are n + 1 objects. Thus some box must contain at least 2 objects. This
combinatorial principle was first used explicitly by Dirichlet (1805-1859). Even though it is extremely simple,
it can be used in many situations, and often in unexpected situations. Note that the principle asserts the
existence of a box with more than one object, but does not tell us anything about which box this might be.
In problem-solving, the difficulty of applying the pigeonhole principle consists in figuring out which are the
‘objects’ and which are the ‘boxes’.
4.4. PIGEON HOLE PRINCIPLE 81

4.4.2 Examples
1. Prove that in a group of three people, there must be two of the same sex.

Solution: There are only n = 2 sexes, but we have n + 1 = 3 people. Here the sexes are the ‘boxes’,
and the people are the ‘objects’. There is no guarantee that two people will both be of the same sex
but if we bring the third person, his/her sex will be the same as one of the two people according to
the pigeonhole principle.
2. Prove that among 13 people, there are at least two who were born in the same month.
Solution: There are n = 12 months (‘boxes’), but we have n+1 = 13 people (‘objects’). Therefore
two or more people were born in the same month.
There are n people present in a room. Where n ≤ 2. Prove that among them there are two
people who have the same number of acquaintances in the room.
Solution: Suppose that there is at most one person (object) in each room (box).

Case 1: When box 0 contains a person.


This would imply that a person from the remaining n-1 people will belong to one of the boxes labeled from
1 to n-2. Since there are n-1 people who will share n-2 boxes, then by pigeon hole principle there will be at
least one box that contains at least 2 objects(people), which contradicts our initial assumption.
Case 2: When box 0 is not occupied.
This would imply that a person from the remaining n people will belong to one of the boxes labeled from 1
to n-1. Then by pigeon hole principle, there will be at least one box that contains at least 2 objects(people),
which contradicts our initial assumption again.

Therefore, those two cases are sufficient to prove that among n people in the room, there must be two
people who have the same number of acquaintances in that room.

4.4.3 Exercises
1. Nobody has more than 300,000 hairs on his head. The capital of Kimisagara has 300,001 inhabitants.
Can you assert with certainity that there are two persons with the same number of hairs on their
heads?
2. Show that given a regular hexagon of side 2 cm and 25 points inside it, there are at least two points
among them which are at most 1 cm distance apart [7]
3. Show that if 7 points are chosen on the circumference or in the interior of a unit circle, such that their
mutual distance apart is greater than or equal to 1, then one of them must be the centre
4. Show that in a party there are always two persons who have shaken hands with the same number of
persons.
5. Prove that, among any 52 integers, two can always be found, such that the difference of their squares,
is divisible by 100.
6. Show that, for any set of 10 points, chosen within a square, whose side is 3 units, there are two points,
in the set, whose distance is at most 2 .
7. There are 7 persons in a group, show that, some two of them, have the same number of acquaintances
among them.
8. Six points are given inside an equilateral triangle of area 4. Prove that among the nine points which
include the three vertices of the triangle and the six given points, three of these form a triangle of area
at most 1.
9. In a tournament with n players, everyone plays with everybody else exactly once. Prove that during
the game there are always two players who have played the same number of games.
82 CHAPTER 4. COMBINATORICS

4.5 GAME THEORY


4.5.1 WINNING STRATEGY
We consider games for two players A and B, who move alternately. A always moves first but otherwise the
rules are the same for A and B. A draw cannot occur. We are given the starting state and the set M of legal
moves. A player loses if he finds himself in a position from which no legal move can be made. We can think
of each position as a vertex of a graph and each move as a directed edge. We consider games with finitely
many vertices and no directed circuit (a position can not repeat). This ensures that one of the players will
lose. The set P of S
all positions
T can be partitioned into the set L of losing positions and the set W of winning
positions: P = L W, L W = 0. A player finding himself in position L will lose provided his opponent
plays correctly. A player finding himself in a position W can force a win whatever his opponent does.
To win, a player must always move so as to force his opponent into a position belonging to L. From each
position in L, every move must result in a position in W. From every position in W, a move to a position in
L must be possible. L must contain at least one final position ƒ from which there is no move out. The player
who leaves his opponent facing such a position has won the game. The problem is to identify the set L of
losing positions.
Most of the following problems can be solved by a simple strategy: Divide the set of all positions into
pairs, so that there is a move from the first to the second element of the pair. Whenever my opponent
occupies one element of a pair, I move to the other element of the pair. Thus, I win, since my opponent
runs out of moves first. Initially, if there is one position without a pair, I should occupy it. Otherwise, I
should be the second player to win. In more complicated games, a table of losing positions should be used
in playing. As a warmup, we will consider some examples with solutions. [8]

4.5.2 Examples
1. Hirwa and Arnold are playing a game where one has to remove either one, two, or three
pens from a box which initially contains 11 pens. The winner is the one who will remove
the last pen. If Hirwa plays first, who has the winning strategy?

Solution: We want to see the one who will remove the 11th pen no matter what. This person will
have to remove the 7th pen because his opponent won’t remove the 11th pen when there are 4 pens in
the box, and after the opponent’s turn, the oppenent will always leave a remainder of 1, 2, or 3 pens
in the box, hence giving a winning position to his fellow. Similarly, for the winner to remove the 7th
pen, he will have to remove the 3rd pen. This means that the one who will start will have a winning
strategy since he has the potential to remove the 3rd pen out of the box no matter what. Therefore,
Hirwa has a winning strategy.

2. Initially there are n checkers on the table. The set of legal moves is the set M = (1, 2,
3,..., k). The winner is the one to take the last checker. Find the losing positions.

The set L consists of all multiples of k + 1. Indeed, if n is not a multiple of k+1, then I can always
move to a multiple of k + 1. My opponent cannot move to the next multiple of k + 1 since he can only
subtract k or less checkers. So he has to move to some number, which is not a multiple of k +1. Then
I simply move into a winning position everytime. Thus, I will finally reach 0, which is also a multiple
of k +1.

3. A and B are going to play a game by turns. Before they start, they form a circle with
2001 other persons. At every turn they can remove one of their neighbors from the circle.
The winner is the one who gets the other person out of the circle. If A starts, decide
who has a winning strategy. Note: The other 2001 persons do not have turns.
4.5. GAME THEORY 83

Solution: When the game starts there are 2001 other persons. This means that A and B divide the
circle in two arcs, one of which has an odd number of persons and the other an even number. The
strategy for A is to remove always a person on the even side. This leaves B with an odd number of
persons on each side. When B plays, A has again a side with an odd number of persons and an even
number on the other, so he can continue with his strategy. B can never hope to win if A plays this
way, since he always has at least one person between himself and A on both sides. Since the game
must end after at most 2001 turns, A wins.

In problems involving games, finding a winning strategy means finding a way to play so that, regardless
of how the other person plays, one is going to win. The key to solve this kind of problems is to find
an invariant in the game and exploit it. The invariant has to be a certain state of the game. We are
looking for a state with the following properties:
• A person in that state cannot win the game.
• If the other person played in that state, we can force him back to that state. This kind of state is
called a losing position. The positions that can send the other player to a losing position are called
winning positions. In the previous example the losing position was having an odd number of persons on
each side. In this type of problems, to find the losing positions it is convenient to look at the positions
near the end of the game when one cannot win and work backwards in the game. It is also a good
strategy to try a few games to look for the invariant.

4.5.3 Exercises
1. Amy and Ben play the game of mis‘ere noughts and crosses on a 3 × 3 square array. On Amy’s turn,
she can place an X in any vacant square, while on Ben’s turn, he can place an O in any vacant square.
The players take turns to place their symbol, with Amy going first. Any player who gets three in a
row (horizontally, vertically or diagonally) immediately loses the game. The game is considered drawn
if there is no winner after all squares have been filled. Which player, if any, has a winning strategy?

2. There are two piles of checkers on a table. A takes any number of checkers from one pile or the same
number of checkers from each pile. Then B does the same. The winner is the one to take the last chip.
Find the losing positions(L).

3. Start with n=2. Two players A and B move alternatetly by adding a proper divisor of n to the current
n. The goal is a number ≥ 1990. Who wins?

4. A and B alternately put white and black knights on the squares of a chessboard, which are unoccupied.
In addition a knight may not be placed on a square threatened by an enemy knight (of the other color).
The loser is the one who cannot move any more. Who wins?

5. A and B alternately draw diagonals of a reqular 1988-gon.They may connect two vertices if the diagonal
does not intersect an earlier one. The loser is the one who cannot move. Who wins?

6. At the start of a game, the numbers 1 and 2 are each written 10 times on a blackboard. Two players
take turns to erase two of the numbers, replacing them with a 1 if they are different and with a 2 if
they are the same. The first player wins if the last number on the board is 1, while the second player
wins if it is 2. Which player has a winning strategy?

7. Two players start with the number 1 and take turns to multiply it by an integer from 2 to 9. The
winner is the first player to obtain a number greater than or equal to 1000. Which player has a winning
strategy?

8. Two people play a game involving n coins on a table. The first player takes at least one, but not all,
of the coins. The players then take turns to take at least one coin, but no more than was taken on
the previous move. The player who takes the last coin is considered the winner. For which values of n
does the second player have a winning strategy?
84 CHAPTER 4. COMBINATORICS

9. Consider a chocolate bar in the shape of an equilateral triangle, with sides of length n, divided by grid
lines into equilateral triangles of side length 1. Two players take turns to break off a triangular piece
along one of the grid lines and pass the remaining block of chocolate to the other player. A player who
is unable to move or who leaves an equilateral triangle of side length 1 is declared the loser. For which
values of n does the second player have a winning strategy?

4.6 PROBLEMS
13
1. If a team won 13 games and lost 7 games, its winning percentage was 13+7 × 100
(a) The Sharks played 10 games and won 8 of these. Then they played 5 more games and won 1 of
these. What was their final winning percentage? Show the steps that you took to find your answer.

(A) 50 (B) 55 (C) 60 (D) 65 (E) 70

(b) The Emus won 4 of their first 10 games. The team played x more games and won all of these.
Their final winning percentage was 70How many games did they play in total? Show the steps that
you took to find your answer.

(A) 11 (B) 15 (C) 13 (D) 20 (E) 18

2. A hat contains six slips of paper numbered from 1 to 6. Amelie and Bob each choose three slips from
the hat without replacing any of the slips. Each of them adds up the numbers on his slips.

(a) Determine the largest possible difference between Amelie’s total and Bob’s total.

(A) 8 (B) 9 (C) 7 (D) 6 (E) 10

(d) If more slips of paper are added to the hat, numbered consecutively from 7 to n, what is the
smallest value of n ¿ 6 so that Amelie and Bob can each choose half of the slips numbered from 1 to n
and obtain the same total?

(A) 8 (B) 9 (C) 7 (D) 6 (E) 10

3. Franco and Sarah play a game four times using the following rules:
(R1) The game starts with two jars, each of which might contain some beans.

(R2) Franco goes first, Sarah goes second and they continue to alternate turns.

(R3) On each turn, the player removes a pre-determined number of beans from one of the jars. If
neither jar has enough beans in it, the player cannot take their turn and loses. If only one jar has
enough beans in it, the player must remove beans from that jar. If both jars have enough beans, the
player chooses one of the jars and removes the beans from that jar.

(R4) Franco must attempt to remove 1 bean on his first turn, 3 beans on his second turn, and 4 beans
on his third turn. On each of his following sets of three turns, Franco must continue to attempt to
remove 1, 3 and 4 beans in sequence.
4.6. PROBLEMS 85

(R5) Sarah must attempt to remove 2 beans on her first turn and 5 beans on her second turn. On
each of her following sets of two turns, Sarah must continue to attempt to remove 2 and 5 beans in
sequence.
(R6) A player is declared the winner if the other player loses, as described in (R3).

(a) At the beginning of the first game, there are 40 beans in one jar and 0 beans in the other jar. After
a total of 10 turns (5 turns for each of Franco and Sarah), what is the total number of beans left in
the two jars?

(A) 11 (B) 12 (C) 13 (D) 14 (E) 15

(b) At the beginning of the second game, there are 384 beans in one jar and 0 beans in the other jar.
The game ends with a winner after a total of exactly n turns. What is the value of n?

(A) 110 (B) 114 (C) 118 (D) 120 (E) 124

(c) At the beginning of the third game, there are 17 beans in one jar and 6 beans in the other jar. There
is a winning strategy that one player can follow to guarantee that they are the winner. Determine
which player has a winning strategy. (A winning strategy is a way for a player to choose a jar on each
turn so that they win no matter the choices of the other player.)

(A) Franco (B) Sarah (C) None of them (D) All of them (E) The question is impossible

(d) At the beginning of the fourth game, there are 2023 beans in one jar and 2022 beans in the other
jar. Determine which player has a winning strategy.

(A) Franco (B) Sarah (C) None of them (D) All of them (E) The question is impossible

4. Al and Bert must arrive at a town 22.5 km away. They have one bicycle between them and must arrive
at the same time. Bert sets out riding at 8 km/h, leaves the bicycle and then walks at 5 km/h. Al
walks at 4 km/h, reaches the bicycle and rides at 10 km/h.
For how many minutes was the bicycle not in motion?

(A) 60 (B) 75 (C) 84 (D) 94 (E) 109

5. A deck of 100 cards is numbered from 1 to 100. Each card has the same number printed on both sides.
One side of each card is red and the other side is yellow. Barsby places all the cards, red side up, on a
table. He first turns over every card that has a number divisible by 2. He then examines all the cards,
and turns over every card that has a number divisible by 3. How many cards have the red side up
when Barsby is finished?

(A) 83 (B) 17 (C) 66 (D) 50 (E) 49

6. There are 90 cards numbered 10 to 99. A card is drawn and the sum of the digits of the number in the
card is noted; if 35 cards are drawn, then, there are at least how many cards whose sum of the digits
are identical?

(A) 7 (B) 4 (C) 3 (D) 5 (E) 9


86 CHAPTER 4. COMBINATORICS

7. Given three points, in the interior of a right angled triangle, At least how many of them are at a
distance not greater than the maximum of the lengths of the sides containing the right angle?
(A) 6 (B) 4 (C) 3 (D) 5 (E) 2

8. Let A be the set of 19 distinct integers, chosen from the Arithmetic Progression 1, 4, 7, 10, . . . , 100.
There should be at least how many distinct integers in A, such that, their sum is 104?

(A) 6 (B) 4 (C) 3 (D) 5 (E) 2


Bibliography

[1] “Byju’s.” https://byjus.com/maths/subsets-of-set-of-real-numbers/.


[2] https://www.math-only-math.com/practice-test-on-sets-and-subsets.html.

[3] https://byjus.com/maths/one-to-one-function/.
[4] https://byjus.com/maths/linear-equations/.
[5] “Cemc.” https://www.cemc.uwaterloo.ca/contests/past_contests.html.
[6] “[john chung] dr john chung s 54 perfect tips for(zlib.org).”

[7] V. T. . V. Seshan, Pathfinder for Olympiad Mathematics. Pearson.


[8] A. Engel, Problem-Solving Strategies. Springer, 1997.

87
88 BIBLIOGRAPHY
Part II

RWANDA MATH OLYMPIAD


GLOSSARY

89
1st edition May 2022

91
Rwanda Math Olympiad Glossary

May 2022

Preface

Glossary is an alphabetical list of words relating to specific dialect with explanations. Rwanda
Math Olympiad (RwMO) Glossary seeks to explain to Rwandan students unfamiliar words and
symbols that are found in math olympiad competitions to boost their interest in mathematics and
problem solving excellence.

Ijambo ry’ ibanze

Inkoranyamagambo ni urutonde rw’ amagambo ajyanye n’ imvugo yihariye hamwe n’ ibisoba-


nuro. Iyi nkoranya magambo igamije gusobanurira abanyeshuri bo mu Rwanda amagambo ata-
menyerewe dusanga mu marushanwa ya olipiyadi (olympiad) y’ imibare, hagamijwe kubongerera
ubumenyi bujyanye n’ imibare no gukora neza ibibazo babajijwe.

Notice: blue-colored words in english definitions section are explained in this glossary.

contacts: [email protected]

Prepared by Arnold Hategekimana Hirwa

92
RwMO Glossary

MATHEMATICAL KINYARWANDA
TERMS ENGLISH DEFINITIONS DEFINITIONS/ TERMS IMAGE/EXAMPLE
the magnitude of a real
number without regard to its
sign. The absolute value of a
number may also be thought as
its distance from zero along ubunini bwumubare nyawo
absolute value real number line. utitaye ku kimenyetso cyawo.

imfuruka iri hagati ya dogere


acute-angle less than 90° 0 na dogere 90

two angles that have a


common side and a common Inguni ebyiri zihujwe n'
adjacent angles vertex. uruhande rumwe.

uburebure bwumurongo wa
perpendicular line from a perpendicular kuva kuri
vertex to the opposite side of a vertex kugera kuruhande
Altitude figure. rwigishushanyo.

the space (usually measured in


degrees) between two
intersecting lines or surfaces at
or close to the point where
angle they meet. imfuruka

a part of a curve, especially a


part of the circumference of a igice cy' umurongo, cyane
arc circle. cyane igice cyumuzingi.

93
RwMO Glossary

a sequence of numbers in urukurikirane rw'imibare aho


which each differs from the buri umwe utandukanywa n'
preceding one by a constant undi hifashishijwe umubare
arithmetic progression quantity. udahinduka. -6,-4,-2,0,2,4,6,...

impuzandengo y'
the average of a set of uruhererekane rw' imibare,
numerical values, as calculated ubarwa nyuma yo guteranya
by adding them together and imibare yose hanyuma
dividing by the number of ukagabanya igiteranyo cyayo
arithmetic(mean) terms in the set. n' ingano y' iyo mibare.

The line that divides


something into two equal Umurongo ugabanya ikintu
bisector parts. mo ibice bibiri bingana.

a board game of strategic skill umukino wibibaho


for two players, played on a byubuhanga bukomeye ku
chessboard on which each bakinnyi babiri, ukinirwa ku
playing piece is moved kibaho cyagenzuwe kuri buri
according to precise rules. The gice cyo gukinisha. Intego ni
object is to put the opponent's ugushyira umwami wuwo
king under a direct attack from bahanganye mu bitero
which escape is impossible bitaziguye aho guhunga
chess (checkmate). bidashoboka (cheque).

ikibaho cya kare


a square board divided into kigabanyijemo ibice mirongo
sixty-four alternating dark and itandatu na bine bisimburana
light squares (conventionally byijimye kandi byerurutse
called ‘black’ and ‘white’), (bisanzwe byitwa 'umukara n'
used for playing chess or umweru'), bikoreshwa mu
chessboard draughts (checkers). gukina chess.

a straight line joining the ends umurongo ugororotse ukora


chord of an arc. ku mpande zombi z' uruziga.

94
RwMO Glossary

a round plane figure whose


boundary (the circumference)
consists of points equidistant
circle from a fixed point (the centre). uruziga

circumcentre centre of circumcircle

a circle touching all the


vertices of a triangle or uruziga rukora kuri buri
circumcircle polygon. nguni y' igishushanyo rurimo.

circumference the distance around a circle. intera ikikije uruziga.

round another, touching it at kuzenguruka ikindi kintu


circumscribe points but not cutting it. ugikoraho ariko utakirenga.
a numerical or constant
quantity placed before and
multiplying the variable in an umubare ukuba ikintu
coefficient algebraic expression. gihinduka.

collinear lying in the same straight line. ibintu biri ku murongo umwe.
Set U = {2,4,6,8,10,12} and
the members of a set or class set A = {4,6,8}, then the
that are not members of a ibigize itsinda rinini bitari mu complement of set A, A′ =
complement (set) given subset. rindi tsinda rito runaka. {2,10,12}

the amount in degrees by ingano muri dogere ibura


which a given angle is less kugira ngo imfuruka
complement (geometry) than 90°. yatanzwe igere kuri 90°. complement of 50° is 40°

a number that is a multiple of umubare utarimo ibice


at least two numbers other ushobora kugabanywa n'
composite number than itself and 1. imibare ibiri itandukanye. 4,8,9,12,25,...

95
RwMO Glossary

internal angle can be more inguni y' imbere ishobora


concave polygon than 180°. kuruta dogere 180°.

inziga zihuje akadomo k'


concentric circles circles with a common centre. impuzandengo.

concurrent (of three or meeting at or tending towards


more lines) one point. guhura kw' imirongo myinshi.

points that lay on the


concyclic points circumference of a circle. utudomo turi ku muzingi.
an arrangement of parts or
elements in a particular form, gutondekanya ibintu mu
configuration figure, or combination. buryo runaka.
a quantity or parameter that
does not change its value
whatever the value of the
variables, under a given set of umubare cyangwa ikintu
constant conditions. kidahinduka.

all interior angles are less than inguni z' imbere zose ziri
convex polygon or equal to 180 degrees munsi ya dogere 180°

itsinda ry' imibare


each of a group of numbers rigaragaragaza akadomo,
used to indicate the position of umurongo, cyangwa ikindi
co-ordinate a point, line, or plane. kintu.

aho imfuruka z' igishushanyo


a polygon having all its ziremerwa haba hakora ku
cyclic polygon vertices lying on a circle. ruziga.

96
RwMO Glossary

the number below the line in a 2 is denominator in this


denominator vulgar fraction; a divisor. icyitarusange fraction: 1/2

a straight line joining two


opposite corners of a square, umurongo uhuza inguni
rectangle, or other straight- ebyiri z' ikinyampande
diagonal sided shape. runaka.

a straight line passing from umurongo ugabanyamo


side to side through the centre uruziga cyangwa ikindi
of a body or figure, especially gishushanyo mo ibice bibiri
diameter a circle or sphere. bingana.
any of the numerals from 0 to
9, especially when forming a umwe mu mibare uhereye
digit part of number. kuri 0 ukageza ku 9. 0,1,2,3,4,5,6,7,8,9

prime numbers without any distinct prime factors of


distinct prime numbers repeats. 9999 are 3, 11 and 101.
umubare wagabanyishijwe
dividend a number that is being divided. undi.
a number that divides another umubare ugabanya undi
divisor without returning a remainder. ntihagire igisaguka.

any of 28 small oblong pieces


marked with 0–6 pips in each kamwe mu mu duce 28
domino half. twerekanwe haruguru.

line segment joining two igice cy' umurongo gihuza


edge (geometry) vertices. utudomo tubiri.
a statement that shows that the
values of two mathematical inyandiko yerekana ko ibintu
equation expressions are equal. bibiri bingana. 2x-2=0

97
RwMO Glossary

having all its sides of the same


equilateral length. impande zose ziba zingana.

two dimensional part of the


euclidean plane euclidean space.

an efficient method for


computing the greatest uburyo bwo gushaka umubare
common divisor (GCD) of two ugabanya imibare ibiri
euclidean algorithm integers (numbers). runaka.

a space in any finite number of


dimensions, in which points
are designated by coordinates
(one for each dimension) and
the distance between two
points is given by a distance
euclidean space formula.

2:2=1; 4:2=2; 12:2=6,...


a number that returns zero as a bivuzeko 2,4,... hakubiyemo
remainder when divided by na 0 ni imibare
even number two. umubare w'imbangikane. y'imbangikane.

excentre centre of excircle


uruziga rukora ku kwaguka
a circle tangent to the kw' impande ebyiri za
extensions of two sides of a mpandeshatu no ku ruhande
excircle triangle and the third side. rwa gatatu.
a number or algebraic
expression that divides another
number or expression
evenly—i.e. with no umubare ugabanya undi
factor remainder. ntihagire igisaguka. 3 and 6 are factors of 12.

98
RwMO Glossary

a set that has a finite number For example, {1,3,5,7} is a


finite set of elements. itsinda rifite ingano. finite set with four elements.
a relation or expression
involving one or more isano iba irimo ikintu kimwe
function variables. cyangwa byinshi bihinduka. y = x + 3
for a given set of two
numbers such as 8 and 1, the
the central number in a geometric mean is equal to
geometric mean geometric progression. √(8×1) = √8 = 2√2

a sequence of non-zero
numbers where each term after
the first is found by urukurikirane rw' imibare itari
multiplying the previous one zeru, aho umubare uba
by a fixed, non-zero number wikubyi inshuro zidahinduka
geometric progression called the common ratio. runaka uwurinyuma. 2,4,8,16,...

the largest integer or the


polynomial of highest degree
that is an exact divisor of each umubare cyangwa ikintu
of two or more integers or kinini gishobora kugabanya
greatest common divisor polynomials. ibintu bibiri.
the intersection of the three
interior angle bisectors of a
incentre triangle.
an inscribed circle of a
polygon that is tangent to each
incircle of the polygon's side.

difference in size, degree, itandukaniro riri hagati y'


inequality circumstances, etc. ibintu runaka.

an infinite ordered set of


infinite sequence numerical quantities. urutonde rutarangira. -3,-2,-1,0,1,2,3,4,5,...
draw (a figure) within another gushushanya mw' imbere y'
so that their boundaries touch ikindi gishushanyo,
inscribe but do not intersect. ugikoraho ariko utakirenga.
a number which is not a
integer fraction; a whole number. umubare utarimo ibice. ...,-2,-1,0,1,2,...

99
RwMO Glossary

having two sides of equal hari impande ebyiri zingana


isosceles length. muri iyo mpandeshatu.
The linear factors of a
polynomial are the first-degree
equations that are the building
blocks of more complex and
linear factors higher-order polynomials.

denoting or relating to a value


or quantity lying at the
midpoint of a frequency
distribution of observed values
or quantities, such that there is
an equal probability of falling icyakabiri cy' umubare
median above or below it. cyangwa ikintu runaka.

a point in the middle of


mid-point something. ni muri kimwe cya kabiri.

a product that we get when


one number is multiplied by
another number; a number that
may be divided by another a
certain number of times igisubizo tubona iyo umubare if we say 4 × 5 = 20, here 20
multiple without a remainder. umwe wikubye undi. is a multiple of 4 and 5
imibare yose itarimo ibice
uhereye kuri rimwe
natural numbers the positive integers. ukazamuka. 1,2,3,4,5,...
the number above the line in a 3 is numerator in this
numerator vulgar fraction. fraction: 3/4

obtuse angle angle greater than 90° inguni iruta dogere 90°
having one left over as a 5 ugabanyije 2 = 2.5 bivuze
remainder when divided by ko 5 ari umubare w'
odd two. Umubare w'igiharwe. igiharwe.

100
RwMO Glossary

the common intersection of the


orthocenter of a triangle three altitudes of a triangle.

A projection of a figure onto a


line or plane so that each
element of the figure is
mapped onto the closest point
orthogonal projection on the line or plane.
bivuze gukora amatsinda
refers to all subsets of a given mato yose agizwe n' imibare
set that contain exactly two cyangwa ibintu bibiri by' the set {1,2,3} all possible
pairwise elements. itsinda rinini runaka. pairs are (1,2),(2,3),(1,3).

If the intersection of two


events is the empty set, then
the events are called pairwise n' igihe amatsinda abiri aba {1, 2, 3} and {4,5,6} are
pairwise disjoint disjoint events. adafite aho ahuriye. pairwise disjoint sets.

(of lines, planes, or surfaces) imirongo cyangwa ibintu


side by side and having the bibiri runaka biba bifite intera
same distance continuously imwe hagati yabyo kuburyo
parallel between them. bidashobora guhura.

Pascal's triangle, in algebra, is


a triangular arrangement of
numbers that gives the
coefficients in the expansion
pascal traingle of any binomial expression.

umubare ushobora
a number that can be kwandikwa nk' umubare
expressed as the square of an umwe wikubye inshuro
perfect square integer. zingana nka wo. 1,4,9,16,25,...

at an angle of 90° to a given


line, plane, or surface or to the gukora imfuruka ya dogere
perpendicular ground. 90° hagati y' ibintu 2 runaka.

101
RwMO Glossary

an expression of more than


two algebraic terms, especially
the sum of several terms that
contain different powers of the
polynomial same variable(s). 2xy-3x
a number that has exactly 2
factors, i.e: divisible only by umubare ugabanywa na
itself and unity. N.B: 1 is not a rimwe cyangwa wo ubwawo
prime number and negative utabariyemo rimwe n' imibare
prime number integers can not be prime. iri munsi ya zeru. 7,11,13,17,...

quadrilateral a four-sided figure ikinyampande enye

the number resulting from the umubare uturuka ku


division of one number by kugabanya umubare umwe
quotient another. ukoresheje undi.

the quantitative relation


between two amounts showing if there is 1 boy and 3 girls
the number of times one value isano iri hagati y' imibare ibiri you could write the ratio as:
contains or is contained within yerekana inshuro umubare 1 : 3 (for every one boy
ratio the other. umwe wikubye uwundi. there are 3 girls).
Any number that can be umubare ushobora
written as a fraction with kugaragazwa n' imibare ibiri,
integers is called a rational aho umwe uba ugabanya
rational number number. undi. -1/30,-7/13,1/2, 1/5, 3/4,...

a quantity that can be


expressed as an infinite imibare yose ibaho harimo n' -22.22565, 3,13.335451,1/3,
real numbers decimal expansion. irimo ibice. √6, 2,...

umubare usigara nyuma yuko


the amount left over when one umubare umwe ugabanyije
remainder quantity is divided by another. undi.

102
RwMO Glossary

uburyo bwo kugaragaza


a mirror image of a shape or ishosha y' kintu uvuye ku
an object, obtained from murongo cyangwa ahantu
reflection flipping the image/object. runaka.

angle that is exactly equal to


right angle 90° inguni ingana nka dogere 90°

the plane figure enclosed by


two radii of a circle or ellipse
sector(circle) and the arc between them.

igice cy' igishushanyo


a part of a figure cut off by a cyatandukanyijwe n' ikindi
segment line or plane intersecting it. hifashishijwe umurogo.

semi-circle a half of a circle. kimwe cya kabiri cy' uruziga.


muri uru rutonde: 2,4,6,8,...
umubare umwe ugenda
a list of things (usually urutonde rw' ibintu urusha uwuri imbere ho
sequence numbers) that are in order. (ubusanzwe ni imibare). imibare ibiri.
The strictly positive integers
are the set defined as: Z>0:=
{x∈Z:x>0} That is, all the
integers that are strictly greater imibare itarimo ibice kandi
strictly positive(integer) than zero: Z>0. isumba zeru. 1,2,3,4,5,......

Itsinda rigizwe n' ibintu biri


a set of which all the elements mu rindi tsinda
subset are contained in another set. ryisumbuyeho.

The central point that splits the akadomo kagabanya ikintu


symmetric point object or shape into two parts. mo ibice bibiri.
a straight line or plane that
touches a curve or curved Umurongo ugororotse ukora
surface at a point, but if ku muzenguruko w' ikintu
extended does not cross it at kiburungushuye (curve,
tangent that point. circle).

103
RwMO Glossary

Ikinyampande enye gifite


a four-sided polygon with at nibura impande ebyiri
trapezium least two sides parellel. ziteganye.
Ikinyampande enye gifite
a four-sided polygon with at nibura impande ebyiri
trapezoid least two sides parellel. ziteganye.

a geometric shape composed imiterere igizwe na kare enye


tetramino of four connected squares. zihuje.

each angular point of a


polygon, polyhedron, or other aho imirongo ibiri ihurira
vertex figure. maze ikarema imfuruka.

104
SYMBOLS

symbol meaning example / explanation


∀ for all n + 2 is odd ∀n is an odd number
∃ exists if x2 + x − 2 = 0, ∃ x in R
ϵ belongs 2n + 1 is odd ∀nϵN

/ does not belong if x2 + 1 = 0 , x ∈R
/
∧ intersects if plane 1 is not parallel to plane 2, then plane 1 ∧ plane 2
| divides 5| 25 and n |n2 + n
̸| does not divide 2̸ | 21
≈ is approximately equal to; is 3.01 ≈ 3 and π≈ 3.14
almost equal to.
̸= is not equal to 2 ̸= 3
∆ triangle we say that ∆ABC is equilateral if all sides are equal
gcd greatest common divisor gcd(17, 170) = 17
LCM lowest common multiple LCM (15,6)=30
IE this means that 2̸ |n + 1 IE n is an odd number
etc and so on 4| n + 2 ∀ n = 2, 6, 10, 14, 18, 22, etc
WLOG without loss of generality for x + y − 2xy > 1 suppose WLOG x ≥ y

L1
line 1 is parallel to line 2
L2

A
B
C AB=BC
A

B C

D angle ACB = angle BCD


∴ Therefore x + 2 = 5 ∴ x=3
Q.E.D. that which was to be demon- it is often placed at the end of a mathematical proof to
strated indicate its completion
∩ intersection A= {a, b, c, d, e}, B= {b, c, f }, A∩B ={b,c}
∪ union A= {a, b, c, d, e}, B= {b, c, f }, A∪B ={a,b,c,d,e,f}
[x, y[ or [x, y) real numbers between x and
y including x but not y.
1
... and so on 3 = 0.3333333...

105
x! the product of all positive in- 4!=4x3x2x1=24
tegers less than or equal to x
⌊x⌋ the greatest integer less than ⌊3.6⌋ = 3; ⌊−2.3⌋ = −3
or equal to x
⌈x⌉ the least integer greater than ⌈3.6⌉ = 4; ⌈−2.3⌉ = −2
or equal to x
⊆ is a subset of A= {a, b, c, d, e}, B= {b, c} IE B⊆ A
⊈ is not a subset of A= {a, b, c, d, e}, B= {f, g, h} IE B⊈ A
∞ infinity
∝ is proportional to a
b = k where k is a positive number, ∴ a ∝ b
: ratio if there are three dogs for every two cats, we would say the
ratio of dog to cats is 3:2.
≡ equivalent to/ is congruent to 3 ≡ 1 mod 2
∅ empty set A= {a, b, c, d, e}, B= {f, g, h}; A∩B = ∅
N natural numbers
Z integers
Q rational numbers
R real numbers
i + ... + k given that nϵZ,
k
X
n (1) 3
X
n=i 2n = 2 + 22 + 23 = 14 (2)
n=1

i × ... × k given that nϵZ,


k
Y
n (3) 3
Y
n=i 2n = 2 × 22 × 23 = 64 (4)
n=1

π the ratio of the circumference π = 3.141592653589793238...


of a circle to its diameter; it
is an irrational number.
∼ similar to ∆ABC ∼ ∆DEF , if their corresponding angles are
congruent and their corresponding sides are in proportion.
⊥ perpendicular t0 If angle between line L1 and line L2 is equal to 90°, L1 ⊥ L2 .
→ implies(if...then...) p:The triangle PQR is isosceles.
q: Two of the angles of the triangle PQR have equal mea-
sure.

√p → q
√ square root 25 = 5

106

You might also like